AH2 Module 5

Réussis tes devoirs et examens dès maintenant avec Quizwiz!

An older client presents to the clinic after a ground level fall at home. What statement by the client indicates the need for more injury prevention education? >"I always take my medicine as directed." >"I only eat little snacks so I don't gain weight." >"I will make sure I drink enough water." >"I make sure to get as much sleep as I used to."

"I only eat little snacks so I don't gain weight." More fall injury prevention education is needed when the client says that he/she will only eat little snacks to prevent weight gain. The brain is sensitive to decreased glucose levels which can lead to falls. This is especially noted in older clients. Taking medication as directed, ensuring adequate hydration, nutrition, and sleep help promote nervous system health and decrease the risk for falls in the elderly.

A client presents to the clinic with a migraine and is lying in a darkened room with a wet cloth on the head after receiving treatment. In preparation for dismissal home, what does the nurse do next? >Allow the client to remain undisturbed. >Assess the client's vital signs. >Remove the cloth because it can harbor microorganisms. >Turn on the lights for a neurologic assessment.

Allow the client to remain undisturbed. The next action by the nurse is to allow the client to remain undisturbed. The client may be able to alleviate pain by lying down in a darkened room with a cool cloth on his or her forehead. If the client falls asleep, he or she would remain undisturbed until awakening. Assessing the client's vital signs, although important, will disturb the client unnecessarily. A cool cloth is helpful for the client with a migraine and does not present enough of a risk that it would be removed. Turning on the lights for a neurologic assessment is not appropriate because light can cause the migraine to worsen.

A client who is recuperating from a spinal cord injury at the T4 level wants to use a wheelchair. What should the nurse teach the client to do in preparation for this activity? a. Push-ups to strengthen arm muscles b. Leg lifts to prevent hip contractures c. Balancing exercises to promote equilibrium d. Quadriceps-setting exercises to maintain muscle tone

Answer: A Arm strength is necessary for transfers and activities of daily living and for use of crutches or a wheelchair, so the nurse should teach the client how to do wheelchair push-ups safely. Equilibrium is not a problem. The client does not have neurologic control of the other activities.

A client has a discectomy and fusion for a herniated nucleus pulposus (HNP). When getting out of bed for the first time, the client reports feeling faint and lightheaded. Which instruction should the nurse provide to the client? a. "Sit upright on the edge of the bed." b. "Slide to the floor to prevent a fall and injury." c. "Bend forward to increase the blood flow to the brain." d. "Lie down immediately so a blood pressure can be obtained."

Answer: A Sitting maintains alignment of the back and allows the nurses to support the client until orthostatic hypotension subsides. Sliding to the floor so the client will not fall and get hurt will induce flexion of the vertebrae, which can traumatize the spinal cord. Because it will increase the blood flow to the brain, bending forward will induce flexion of the vertebrae, which can traumatize the spinal cord. Rapid movement can flex the vertebrae, which can traumatize the spinal cord; taking the blood pressure at this time is not necessary.

What action should the nurse take to prevent precipitating a painful attack in a client with tic douloureux? a. Avoid walking swiftly by the client. b. Keep the client in the prone position. c. Discontinue oral hygiene temporarily. d. Massage both sides of the face frequently.

Answer: A The nurse should avoid walking swiftly past the client because drafts or even slight air currents can initiate pain. The client may assume any position of comfort, but pressure on the face while in the prone position may trigger an attack. Although the procedure for oral hygiene may be modified, it is not discontinued. Massaging may trigger an attack and should be avoided.

The registered nurse (RN) is teaching a student nurse about taking a current history from a client who is suspected to have a neurologic disorder. Which statement made by the student nurse indicates the RN needs to follow up? Select all that apply. a. "I should ask the client about insomnia." b. "I should ask the client about diabetes mellitus." c. "I should ask the client about weakness and clumsiness." d. "I should ask the client about any difficulty in swallowing." e. "I should ask the client about numbness and tingling sensations."

Answer: A, B The nurse should ask a client about insomnia and diabetes mellitus while documenting that client's past medical history, not during the current history. Therefore the RN will follow up to correct this misconception. All the other statements are correct. While taking and documenting the current history of a client, the nurse should ask the client about any feeling of weakness and clumsiness in the body, swallowing difficulties, and numbness and tingling sensations. These may help the nurse to identify the client's current health problems.

Carbidopa-levodopa is prescribed for a client with Parkinson disease. The nurse assesses for which adverse responses that are associated with this medication? Select all that apply. a. Nausea b. Lethargy c. Bradycardia d. Polycythemia e. Emotional changes

Answer: A, C Nausea and vomiting may occur; this reflects a central emetic reaction to levodopa. Changes in affect, mood, and behavior are related to toxic effects of carbidopa-levodopa. Insomnia, tremors, and agitation are side effects that may occur, not lethargy. Tachycardia and palpitations, not bradycardia, occur. Anemia and leukopenia, not polycythemia, are adverse reactions.

A recently hospitalized client with multiple sclerosis is concerned about generalized weakness and fluctuating physical status. What is the priority nursing intervention for this client? a. Encourage bed rest b. Space activities throughout the day c. Teach the limitations imposed by the disease d. Have one of the client's relatives stay at the bedside

Answer: B Spacing activities will encourage maximum functioning within the limits of strength and fatigue. Bed rest and limited activity may lead to muscle atrophy and calcium depletion. Strengths, rather than limitations, should be stressed. Having one of the client's relatives stay at the bedside is unnecessary. It is the nurse's responsibility to maintain client safety and meet client needs.

A client who recently experienced a brain attack (cerebrovascular accident, CVA) and who has limited mobility reports constipation. What is most important for the nurse to determine when collecting information about the constipation? a. Presence of distention b. Extent of weight gained c. Amount of high-fiber food consumed d. Length of time this problem has existed

Answer: D First, the nurse should establish when the client last defecated because the client may have perceived constipation. Abdominal distention may or may not be observed with constipation. Weight gain has no relationship to constipation. Although lack of bulk in the diet can lead to constipation, particularly in clients with limited activity or an inadequate fluid intake, the lack of bulk in the diet is not the most significant information to obtain at this time.

A client with a traumatic brain injury from a motor vehicle crash is monitored for signs/symptoms of increased intracranial pressure (ICP). Which sign/symptoms does the nurse monitor for? >Changes in breathing pattern >Dizziness >Increasing level of consciousness >Reactive pupils

Changes in breathing pattern The nurse monitors for changes in breathing pattern. This may be indicative of increased intracranial pressure secondary to compression of areas of the brain responsible for respiratory control. Dizziness is a symptom of brain injury, not increased intracranial pressure. Increasing level of consciousness and reactive pupils are desired outcomes for this client.

A client who had a brain attack (cerebrovascular accident, CVA) two weeks ago is having problems communicating. The nurse shows the client a picture of a baseball and asks the client to identify it and its characteristics. The client describes its color, size, and purpose but cannot identify it as a ball. How will the nurse document this finding in the client's medical record? a. Anomia b. Apraxia c. Dysarthria d. Dysphagia

Answer: A Anomia Clients with anomia cannot remember names of objects. Clients with apraxia cannot use objects properly or complete sequential movement. Clients with dysarthria know what they want to say but cannot speak clearly because there is motor impairment caused by a central or peripheral nervous system injury. Clients with dysphagia have difficulty swallowing; they do not have speech problem.

When caring for a client with a head injury that may have involved the medulla, the nurse bases assessments on the knowledge that the medulla controls a variety of functions. Which functions will the nurse assess? Select all that apply a. Balance b. Breathing c. Pulse rate d. Fat metabolism e. Temperature regulation

Answer: B, C The medulla, part of the brainstem just above the foramen magnum, is concerned with vital functions such as respirations. The medulla is concerned with vital functions such as the heart rate. The cerebellum controls balance. Fat metabolism is not controlled by the central nervous system. Temperature regulation is controlled by the hypothalamus.

A client has a tonic-clonic seizure caused by an overdose of aspirin. What is an appropriate nursing action? a. Check reflexes every 2 hours b. Prepare a setup for a central venous pressure (CVP) line c. Insert a urinary retention catheter d. Monitor vital signs every 15 minutes

Answer: D Because of the lethal toxicity of an aspirin overdose, hypotensive crisis and cardiac irregularities can occur. The central nervous system is not involved at the reflex level at this time. CVP readings are not indicated in this situation. Inserting a urinary retention catheter is not the priority at this time.

The nurse is reviewing the history of a client who has been prescribed topiramate (Topamax) for prevention of migraines. The nurse plans to contact the primary care provider (PCP) if the client has which condition? >Bipolar disorder >Diabetes mellitus >Glaucoma >Hypothyroidism

Bipolar disorder The nurse contacts the PCP after reviewing the history of a client with bipolar disorder who has been prescribed topiramate. Cases of suicide have been associated with topiramate when it is used in larger doses of 400 mg daily, most often in clients with bipolar disorder. Topiramate is not contraindicated in clients with diabetes mellitus, glaucoma, or hypothyroidism.

A client has just returned from cerebral angiography. Which symptom does the client display that causes the nurse to act immediately? >Bleeding >Increased temperature >Severe headache >Urge to void

Bleeding After a cerebral angiography, the nurse would immediately react if the client had any bleeding. If bleeding is present at the puncture site, manual pressure on the site is maintained along with immediate notification of the primary care provider. Increased temperature or the urge to void are not typical complications of cerebral angiography. Severe headache is a typical complication of a lumbar puncture, but not of cerebral angiography.

The nurse has just received report on a group of clients. Which client does the nurse assess first? >Client who was in a car accident and has a Glasgow Coma Scale score of 14 >Client who had a cerebral arteriogram and has a cool, pale leg >Client who has a headache after undergoing a lumbar puncture >Client who has expressive aphasia after a left-sided stroke

Client who had a cerebral arteriogram and has a cool, pale leg The nurse first assesses the client with a cool, pale leg after an arteriogram. This assessment finding could indicate clot formation at the catheter insertion site and loss of blood flow to the extremity. The client with a GCS of 14, the client with a headache following a lumbar puncture, and the client with expressive aphasia need to be assessed as soon as possible.

Which change in the cerebrospinal fluid (CSF) indicates to the nurse that a client may have bacterial meningitis? >Cloudy, turbid CSF >Decreased white blood cells >Decreased protein >Increased glucose

Cloudy, turbid CSF Cloudy, turbid CSF indicates to the nurse that the client may have bacterial meningitis. Clear fluid is a sign of viral meningitis. Increased white blood cells, increased protein, and decreased glucose are signs of bacterial meningitis.

A client recovering from a stroke reports double vision that is preventing the client from effectively completing activities of daily living. How does the nurse help the client compensate? >Approaches the client on the affected side >Covers the affected eye >Encourages turning the head from side to side >Places objects in the client's field of vision

Covers the affected eye The nurse helps the stroke client compensate with double vision by covering the affected eye. Covering the client's affected eye with a patch prevents diplopia. The client who is recovering from a stroke would always be approached on the unaffected side. The nurse may encourage side-to-side head turning for clients with hemianopsia (blindness in half of the visual field). Objects would be placed in the field of vision for the client with a decreased visual field.

A client has Guillain-Barré syndrome. Which interdisciplinary health care team members does the nurse plan to collaborate with to help prevent pressure ulcers related to immobility in this client? Select all that apply. >Certified hospital chaplain >Family members >Dietitian >Occupational therapist (OT), Social worker

Family members Dietitian Occupational therapist (OT), Social worker The nurse plans to collaborate with family members, the dietician, and OT to help prevent pressure ulcers in the client with GBS. Family members would help to develop interventions to prevent these ulcers, because the family will mostly likely be directly involved in the client's care. Malnutrition puts the client at greater risk for pressure ulcers, so the dietitian must be included as well. The OT can provide assistive devices that will help prevent ulcers. The certified hospital chaplain and the social worker can assist with providing additional psychosocial support but would not be involved with direct prevention of ulcers. The social worker would also assist with the discharge plan and reintegration into the community.

In the emergency department (ED), which is the nursing priority in assessing the client with a spinal cord injury? >Indication of allergies >Level of consciousness >Loss of sensation >Patent airway

Patent airway The nursing priority when assessing a client with a spinal cord injury is a patent airway. Clients with injuries at or above T6 are at risk for respiratory complications. Assessing for a patent airway is essential. Asking the client about current medications and allergies is part of every trauma assessment. Assessing the level of consciousness utilizing the Glasgow Coma Score (GCS) is an important part of the trauma assessment. Determining the level of loss of sensation will be included in the neurological evaluation.

Which nursing intervention is best for preventing complications of immobility when caring for a client with spinal cord problems? >Encouraging nutrition >Frequent ambulation >Regular turning and repositioning >Special pressure-relief devices

Regular turning and repositioning Regular turning and repositioning are the best way to prevent complications of immobility in clients with spinal cord problems. A registered dietitian may be consulted to encourage nutrition to optimize diet for general health and to reduce osteoporosis. Frequent ambulation may not be possible for these clients. Use of special pressure-relief devices is important but is not the best way to prevent immobility complications.

In assessing a client with back pain, the nurse uses a paper clip bilaterally on each limb. What is the nurse assessing? >Gait >Mobility >Sensation >Strength

Sensation The nurse uses a paper clip bilaterally on each limb of the client with back pain to assess sensation. Both extremities may be checked for sensation by using a paper clip and a cotton ball for comparison of light and deep touch. The client may feel sensation in both limbs but may experience a stronger sensation on the unaffected side. Gait is assessed by having the client walk. Mobility is assessed by determining the client's ability to move on his/her own, turn or perform ADLs. Strength is measured by having the client perform bilateral grips.

A client newly diagnosed with Parkinson disease (PD) is being discharged. Which instruction is best for the nurse to provide to the client's spouse? >Administer medications promptly on schedule to maintain therapeutic drug levels. >Complete activities of daily living for the client. >Provide high-fiber, high-carbohydrate foods. >Speak loudly for better understanding.

Administer medications promptly on schedule to maintain therapeutic drug levels. Administering medications promptly on schedule is a correct statement. The best instruction the nurse can give to the spouse of a PD client about to be discharged is to give schedule medications promptly in order to keep drug levels therapeutic.

A client with trigeminal neuralgia is admitted for a percutaneous stereotactic rhizotomy in the morning. The client currently reports pain. What does the nurse do next? >Administers pain medication as requested >Ensures that the client has nothing by mouth (NPO) >Ensures that the preoperative laboratory work is complete >Performs a preoperative assessment

Administers pain medication as requested The next action the nurse needs to do is to give pain medication to the preoperative client with trigeminal neuralgia who is complaining of pain. Addressing the client's pain is the priority nursing intervention because pain is the main symptom of trigeminal neuralgia. After the client's pain has been addressed, the preoperative assessment can be completed, questions and concerns can be addressed, and any further testing can be completed. This client is not required to be NPO until after midnight.

After a hurricane, the nurse is assessing the response of a client to stimuli on the Glasgow Coma Scale (GCS) as a part of the primary survey. The nurse observes that the client opens his eyes when his name is stated, uses disorganized words, and is unable to follow commands, but attempts to remove the offending stimulus. What is the Glasgow coma score for this client? Record your answer using a whole number. a. _________________________

Answer: 11 Glasgow scale (GCS) is used by the nurse to conduct neurologic assessment as a part of primary survey. It is performed to determine the client's response to verbal and/or painful stimuli in order to assess the level of consciousness and degree of disability. A score of 3 is given when the client opens the eyes when the name is stated. If disorganized use of words is present, a score of 3 is given. A score of 5 is given when there is a lack of obedience but attempts to remove the offending stimulus. Therefore the client's GCS score would be 3+3+5= 11.

After a client is treated for a spinal cord injury, the healthcare provider informs the family that the client is a paraplegic. The family asks the nurse what this means. Which explanation should the nurse provide? a. Lower extremities are paralyzed. b. Upper extremities are paralyzed. c. One side of the body is paralyzed. d. Both lower and upper extremities are paralyzed.

Answer: A Both legs and generally the lower part of the body are paralyzed in paraplegia. There is no term to describe only upper extremities affected; all parts below an injury are affected. One side of the body paralyzed describes hemiplegia. The paralysis of both lower and upper extremities describes quadriplegia.

A client with multiple sclerosis is in remission. Which diversional activity should the nurse encourage that best meets the client's needs while in remission? a. Hiking b. Swimming c. Sewing classes d. Watching television

Answer: B Swimming helps keep the muscles supple, without requiring fine-motor activity. Hiking might prove too rigorous for the client. Sewing requires fine-motor activity and will be difficult for the client. Sedentary activities are not helpful in maintaining muscle tone.

The nurse provides a client with left-sided weakness with instructions on how to safely use a cane. The nurse should demonstrate proper use of the cane by holding it where? a. On alternating sides b. On the right side c. On the side of the weakness d. On the side of the client's choice

Answer: B The cane should be used on the stronger (unaffected) side of the body to add strength, decrease dependence on the weaker (affected) side, and aid in balance during ambulation. Correct use of a cane does not involve alternating sides, using the cane on the affected (weaker) side, or using the side of the client's choice.

The nurse is teaching a client about the risk factors of restless legs syndrome. Which statement by the client indicates a correct understanding of the nurse's instruction? >"Cigarettes and alcohol must be avoided." >"I need to exercise my legs before bedtime." >"It is important to stay off my feet." >"Over-the-counter drugs must not be taken."

"Cigarettes and alcohol must be avoided." The correct statement about the risks of restless legs syndrome is cigarettes and alcohol must be avoided. Clients with restless legs syndrome need to avoid as many risk factors as possible or make lifestyle modifications. Examples include avoiding caffeine and alcohol, quitting smoking, and losing weight. Clients with RLS need to be encouraged to exercise but not engage in strenuous activity within 2-3 hours before bedtime. Use of over-the-counter drugs is not contraindicated for clients with restless legs syndrome.

A client is admitted with a spinal cord injury at the seventh cervical vertebra secondary to a gunshot wound. Which nursing intervention is the priority for this client's care? >Auscultating bowel sounds every 2 hours >Beginning a bladder retraining program >Monitoring nutritional status >Positioning the client to maximize ventilation potential

Positioning the client to maximize ventilation potential The priority nursing intervention for a client with a spinal cord injury at the seventh cervical vertebra is to position the client to maximize ventilation potential. Airway management is the priority for the client with a spinal cord injury. The client with a cervical spinal cord injury is at high risk for respiratory compromise because the cervical spinal nerves (C3-C5) innervate the phrenic nerve, controlling the diaphragm. Auscultating bowel sounds is important since paralytic ileus can develop from a SCI; however this is not the priority intervention. Beginning bladder retraining and monitoring the nutritional status will be important for adequate healing and progress to rehabilitation. However, these interventions can be delayed until major life threats are addressed.

A client with dementia and Alzheimer's disease is discharged to home. The client's daughter says, "He wanders so much, I am afraid he'll slip away from me." What resource does the nurse suggest? >Alzheimer's Wandering Association >Lost Family Members Tracking Association >National Alzheimer's Group >Safe Return Program

Safe Return Program The discharge nurse suggests the Safe Return Program to the daughter of a client who wanders at home. The Safe Return Program, a national, government-funded program of the Alzheimer's Association assists in the identification and safe, timely return of those with dementia who wander off and become lost. The Alzheimer's Wandering Association, National Alzheimer's Group, and Lost Family Members Tracking Association do not exist.

Which cranial nerve allows a person to feel a light breeze on the face? >I (olfactory) >III (oculomotor) >V (trigeminal) >VII (facial)

V (trigeminal) Cranial nerve V (trigeminal) allows the person to feel a light breeze on the face. This nerve is responsible for sensation from the skin of the face and scalp and the mucous membranes of the mouth and nose. Cranial nerve I (olfactory) is responsible for smell. Cranial nerve III (oculomotor) is responsible for eye movement. Cranial nerve VII (facial) is responsible for pain and temperature from the ear area, deep sensations from the face, and taste from the anterior two thirds of the tongue.

The nurse is caring for a client who is scheduled to have a transcranial Doppler (TCD). What does this diagnostic test evaluate? >Cerebral vasospasm >Cerebrospinal fluid >Evoked potentials >Intracranial pressure

Cerebral vasospasm A transcranial Doppler (TCD) is used to evaluate cerebral vasospasm or narrowing of arteries. It is noninvasive. Cerebrospinal fluid is obtained and measured during a lumbar puncture (LP). Evoked potentials measure the electrical signals in the brain during an EEG. Intracranial pressure is a measurement of blood, brain tissue, and cerebral spinal fluid and is not measured by TCD.

Question 12 of 20 The nurse admits a client with suspected myasthenia gravis (MG). The nurse anticipates that the primary health care provider (PHCP) will request which medication to aid in the diagnosis of MG? >Atropine >Edrophonium chloride (Tensilon) >Methylprednisolone (Solu-Medrol) >Ropinirole (Requip)

Edrophonium chloride (Tensilon) The nurse expects the PHCP to request edrophonium chloride for a newly admitted client suspected of having MG. Edrophonium chloride (Tensilon) and neostigmine bromide (Prostigmin) may be used for testing for MG. Tensilon is used most often because of its rapid onset and brief duration of action. This drug inhibits the breakdown of acetylcholine (ACh) at the postsynaptic membrane, which increases the availability of ACh for excitation of postsynaptic receptors.Atropine has parasympatholytic effects and is the antidote for edrophonium chloride. Methylprednisolone (Solu-Medrol) is a glucocorticoid that is used to treat inflammatory disorders. Ropinirole (Requip) is a dopamine agonist used in the treatment of restless leg syndrome (RLS).

The nurse is caring for a client with Guillain-Barré syndrome (GBS) who is receiving intravenous immunoglobulin (IVIG). Which assessment finding warrants immediate evaluation? >Chills >Generalized malaise >Headache with stiff neck >Temperature of 99° F (37° C)

Headache with stiff neck Immediate evaluation is needed when a client with GBS receiving IVIG complains of a headache with stiff neck. This may be a sign of aseptic meningitis, a possible serious complication of IVIG therapy. Chills, generalized malaise, and a low-grade fever are minor adverse effects of IVIG therapy and do not indicate that the therapy must be stopped.

The nurse is caring for a client diagnosed with partial seizures after encephalitis, who is to receive carbamazepine (Tegretol). The nurse plans to monitor the client for which adverse effects? Select all that apply. >Alopecia >Headaches >Dizziness >Diplopia >Increased blood glucose

Headaches Dizziness Diplopia Adverse effects the nurse must monitor for in a client taking carbamazepine for partial seizures after encephalitis include: headaches, dizziness, and diplopia. Carbamazepine affects the central nervous system, although it's mechanism of action is unclear. Carbamazepine does not cause alopecia and does not increase blood glucose. Divalproex (Depakote) and valproic acid (Depakene) may cause alopecia.

A client with Parkinson disease (PD) is being discharged home with his wife. To ensure success with the management plan, which discharge action is most effective? >Involving the client and his wife in developing a plan of care >Setting up visitations by a home health nurse >Telling his wife what the client needs >Writing up a detailed plan of care according to standards

Involving the client and his wife in developing a plan of care The discharge plan most effective when discharging a client home with his spouse is to involve both the client and his wife in developing the plan of care. Involving the client and spouse in drawing up a plan of care is the best way to ensure success with the management plan. Home health nurse visitations are generally helpful but may not be needed for this client. The management plan must be collaborative and include not only the spouse but the client to ensure buy-in. Evidence-based guidelines would be utilized.

A client who has just undergone spinal surgery must be moved. How does the nurse plan to move this client? >Getting the client up in a chair >Keeping the client in the Trendelenburg position >Lifting the client in unison with other health care personnel >Log rolling the client

Log rolling the client Log rolling the client who has undergone spinal surgery is the best way to keep the spine in alignment. The client who has undergone spinal surgery must remain straight and turned as a unit. The Trendelenburg position is not indicated for the client who has undergone spinal surgery, nor should the client be lifted or encouraged to get up in a chair.

The nurse is about to administer a contrast medium to the client undergoing diagnostic testing. Which question does the nurse first ask the client? >"Are you in pain?" >"Are you taking ibuprofen daily >"Are you wearing any metal?" >"Do you know what this test is for?"

"Are you taking ibuprofen daily" The first question the nurse asks is if the client uses Ibuprofen on a daily basis. Ibuprofen is an NSAID, and daily use may place the client's renal function at risk. The client would also be asked about allergies to contrast agents, daily use of Metformin, and any conditions that may compromise kidney function. Inquiring if a client is in pain is always part of nursing assessment but would not be the first question to ask. The nurse would use this opportunity for education to confirm the client knows the reason for the test and take this time to answer any questions. Diagnostic testing involving magnetic resonance imaging, not contrast medium, requires precautions around metal objects.

The nurse is performing a neurologic assessment on an 81-year-old client. Which physiologic change does the nurse expect to find because of the client's age? >Decreased coordination >Increased sleeping during the night >Increased touch sensation >Nightly confusion

Decreased coordination When performing a neurologic assessment on an elderly client, the nurse expects to find decreased coordination. Older adults experience decreased coordination as a result of the aging process. Older adults frequently go to bed earlier and arise earlier than younger adults. Sensation to touch is decreased not increased. Nightly confusion, sometimes referred to as "sundowning," is not an expected change with all older adults.

The daughter of a client who has had a stroke asks the nurse for additional resources. What is the nurse's best response? >"Call hospice." >"Check the Internet." >"The National Stroke Association has resources available." >"The charge nurse at the desk has all of the information."

"The National Stroke Association has resources available." The nurse's best response about additional resources for stroke is the National Stroke Association. The National Stroke Association is a specific and reliable resource that can be recommended. Additional resources are frequently provided as part of the discharge teaching the nurse will provide. Hospice care is appropriate for clients who are terminally ill, not a client who has had a stroke necessarily. Sources on the Internet may be very broad and unreliable or lack evidence to support their recommendations. The role of the client's nurse is to advocate for the client and not to refer all questions to the charge nurse.

A client is being evaluated for signs associated with myasthenic crisis or cholinergic crisis. Which symptoms lead the nurse to suspect that the client is experiencing a cholinergic crisis? >Abdominal cramps, blurred vision, facial muscle twitching >Bowel and bladder incontinence, pallor, cyanosis >Increased pulse, anoxia, decreased urine output >Restlessness, increased salivation and tearing, dyspnea

Abdominal cramps, blurred vision, facial muscle twitching The nurse suspects a cholinergic crisis when the client experiences abdominal cramps, blurred vision, and facial muscle twitching. These are signs of an acute exacerbation of muscle weakness symptoms of cholinergic crisis caused by overmedication with cholinergic (anticholinesterase) drugs. Bowel and bladder incontinence, pallor, cyanosis, increased pulse, anoxia, and decreased urine output are symptoms indicating a myasthenic crisis. Restlessness, increased salivation and tearing, and dyspnea are symptoms indicating a mixed myasthenic-cholinergic crisis.

The nurse is evaluating the collaborative care of a client with traumatic brain injury (TBI). What is the most important goal for this client? >Achieving the highest level of functioning >Increasing cerebral perfusion >Preventing further injury >Preventing skin breakdown

Achieving the highest level of functioning The most important nurse's goal for the client with TBI is to help him or her achieve the highest level of functioning possible. The nurse assesses cerebral perfusion, such as oxygenation status, but cannot increase cerebral perfusion. Prevention of injury from falls, infection, or further impairment of cerebral perfusion is part of a larger goal for this client. Prevention of skin breakdown is a goal for the care of any client.

During the neurologic assessment of a client with a tentative diagnosis of Guillain-Barré syndrome, what does the nurse expect the client to manifest? a. Diminished visual acuity b. Increased muscular weakness c. Pronounced muscular atrophy d. Impairment in cognitive reasoning

Answer B Increased muscular weakness Muscular weakness with paralysis results from impaired nerve conduction because the motor nerves become demyelinated. Diminished visual acuity usually is not a problem; motor loss is greater than sensory loss, with paresthesia of the extremities being the most frequent sensory loss. Demyelination occurs rapidly early in the disease, and the muscles will not have had time to atrophy; this can occur later if rehabilitation is delayed. Only the peripheral nerves are involved; the central nervous system is unaffected.

A healthcare provider determines that a client has myasthenia gravis. Which clinical findings does the nurse expect when completing a health history and physical assessment? Select all that apply. a. Double vision b. Problems with cognition c. Difficulty swallowing saliva d. Intention tremors of the hands e. Drooping of the upper eyelids f. Nonintention tremors of the extremities

Answer: A, C, E Double vision occurs as a result of cranial nerve dysfunction. Facial muscles innervated by the cranial nerves often are affected; difficulty with swallowing (dysphagia) is a common clinical finding. Drooping of the upper eyelids (ptosis) occurs because of cranial nerve III (oculomotor) dysfunction. Myasthenia gravis is a neuromuscular disease with lower motor neuron characteristics, not central nervous system symptoms. Intention tremors of the hands are associated with multiple sclerosis. Nonintention tremors of the extremities are associated with Parkinson disease.

Which central nervous system manifestation observed in a client with a respiratory disorder indicates inadequate oxygenation? a. Late cyanosis b. Early tachypnea c. Late use of accessory muscles d. Early unexplained restlessness

Answer: D Early unexplained restlessness is a central nervous system sign of inadequate oxygenation that may be observed in the client with a respiratory disorder. Late cyanosis is a cardiovascular sign of inadequate oxygenation. Early tachypnea and late use of accessory muscles are signs of inadequate oxygenation associated with the respiratory system.

A client has a tonic-clonic seizure. Which is the priority nursing intervention during the tonic-clonic stage of the seizure? a. Go for additional help b. Establish a patent airway c. Turn the client on the side d. Protect the client from injury

Answer: D Protecting the client from injury, together with observation and documentation of the seizure activity, is the primary nursing care for a client with a tonic-clonic seizure. The client should not be left unattended. Establishing a patent airway is done after the seizure; the mouth should not be pried open to insert an airway during a seizure because injury may occur. Turning the client on the side will assist with establishing an airway after the seizure, but it is an unsafe action during a seizure.

While assessing the airway patency of a client after a bomb blast, the nurse suspects severe brain injury and gives a score of 7 using the Glasgow Coma Scale (GCS). Which intervention is most appropriate for the client? a. Performing the jaw-thrust maneuver b. Maintaining vascular access using a large-bore catheter c. Observing for chest wall trauma or other physical abnormalities

Answer: D The most appropriate intervention for a client with a GCS score of 7 is preparing for endotracheal intubation and mechanical ventilation. The jaw-thrust maneuver is performed in a client if there is any risk of spinal injury. The use of large-bore catheters to maintain vascular access is done to perform resuscitation in traumatic conditions. Observing for chest wall trauma or other physical abnormalities may not be the appropriate intervention for a client with brain injury.

The wife of a client with Alzheimer's disease mentions to the home health nurse that, although she loves him, she is exhausted caring for her husband. What does the nurse do to alleviate caregiver stress? >Arranges for respite care >Provides positive reinforcement and support to the wife >Restrains the client for a short time each day, to allow the wife to rest >Teaches the client improved self-care

Arranges for respite care The home health nurse can help relieve caregiver stress for the wife caring for her husband with Alzheimer's disease by arranging for respite care for the wife. Respite care can give the wife some time to reenergize and will provide a social outlet for the client. Providing positive reinforcement and support is important but does not help provide a solution to the wife's situation. Restraints are almost never appropriate and are used only as a last resort. The client with Alzheimer's disease typically is unable to learn improved self-care.

A client is being discharged to home with progressing stage I Alzheimer's disease. The family expresses concern to the nurse about caring for their parent. What is the priority for best continuity of care? >Assigning a case manager >Ensuring that all family questions are answered before discharge >Providing a safe environment >Referring the family to the Alzheimer's Association

Assigning a case manager The priority for the best continuity of care for a client about to be discharged with progressing Stage I Alzheimer's disease is to assign a case manager to the client and family. Whenever possible, the client and family need the services of a case manager who can assess their needs for health care resources and facilitate appropriate placement throughout the continuum of care. Continuity of care is important through all stages of Alzheimer's disease. Ensuring all questions are answered and providing a safe environment are necessary for family support. The Alzheimer's Association will also be able to help provide information and support to the family.

The nurse is caring for a client with advanced Alzheimer's disease. Which communication technique is best to use with this client? >Assuming that the client is not totally confused >Providing the client with several options to choose from >Waiting for the client to express a need >Writing down instructions for the client

Assuming that the client is not totally confused The best communication technique to use for a client with advanced Alzheimer's disease is to not assume that the client is totally confused and cannot understand what is being said. Choices need to be limited. Too many choices cause frustration and increased confusion in the client. Rather than waiting for the client to express a need, try to anticipate the client's needs and interpret nonverbal communication. Just writing down instructions may be confusing for the client. It is better to provide the client instructions with pictures, and put them in a highly visible place.

The nurse encourages a ventilated client with advanced Guillain-Barré syndrome (GBS) to communicate by which simple technique? >Blinking for "yes" or "no" >Moving lips to speak >Using sign language >Using a laptop to write

Blinking for "yes" or "no" To communicate, a ventilated client with advanced GBS needs to blink for "yes" or "no." A simple technique involving eye blinking or moving a finger to indicate "yes" and "no" is the best way for the ventilated client with GBS to communicate. Moving the lips is difficult to do around an endotracheal tube and is exhausting for the client. Sign language is very time-consuming to learn, unless the client and family already know it. Use of a laptop may prove too challenging for the client in advanced stages of GBS.

A client is admitted into the emergency department (ED) with frontal-temporal pain, preceded by a visual disturbance. The client is upset and thinks it is a stroke. What does the nurse suspect may be occurring? >Classic migraine >Meningitis >Stroke >West Nile virus

Classic migraine The nurse suspects that a classic migraine could be present when an ED client complains of frontal-temporal pain preceded by a visual disturbance. These symptoms are most typical of a classic migraine. Meningitis may present with a headache and visual disturbance but is usually accompanied by nuchal rigidity (neck stiffness) and fever. The symptoms of stroke will vary depending upon the area affected. Mild cases of West Nile virus may be asymptomatic or present with flu-like symptoms, whereas severe cases may lead to loss of consciousness and death.

The nurse has just received report on a group of clients on the neurosurgical unit. Which client is the nurse's first priority? >Client whose deep tendon reflexes have become hyperactive >Client who displays plantar flexion when the bottom of the foot is stroked >Client who consistently demonstrates decortication when stimulated >Client whose Glasgow Coma Scale (GCS) has changed from 15 to 13.

Client whose Glasgow Coma Scale (GCS) has changed from 15 to 13. After receiving report on a group of clients, the nurse's first priority is to assess the client whose GCS has changed from 15 to 13. A decrease of 2 or more points in the Glasgow Coma Scale total is clinically significant and indicates a major change in neurologic status. This finding must be reported immediately to the primary health care provider (PHCP).The client with hyperactive reflexes, the client displaying plantar flexion when the bottom of the foot is stroked, and the client with decortication upon stimulation will need to be assessed, but they do not require immediate attention.

The nurse is reviewing the medication history of a client diagnosed with myasthenia gravis (MG) who has been prescribed a cholinesterase (ChE) inhibitor. The nurse contacts the primary health care provider (PHCP) if the client is taking which medication? >Acetaminophen (Tylenol) >Diazepam (Valium) >Furosemide (Lasix) >Ibuprofen (Motrin)

Diazepam (Valium) The nurse contacts the PHCP if the client with MG who has been prescribed a ChE is also taking diazepam. Diazepam (Valium) would be avoided because it may increase the client's weakness. Acetaminophen (Tylenol) is an analgesic and antipyretic. It does not interact with ChE inhibitors. Furosemide (Lasix) is a diuretic and does not interact with ChE inhibitors. Ibuprofen (Motrin) is a nonsteroidal analgesic and does not interact with ChE inhibitors.

A client with severe muscle spasticity has been prescribed tizanidine (Zanaflex, Sirdalud). The nurse instructs the client about which adverse effect of tizanidine? >Drowsiness >Hirsutism >Hypertension >Tachycardia

Drowsiness Adverse effects of tizanidine include drowsiness and sedation. Tizanidine (Zanaflex, Sirdalud) is a centrally acting skeletal muscle relaxant. It does not cause hirsutism, hypertension, or tachycardia.

A client will be receiving plasmapheresis for treatment of Guillain-Barre'syndrome (GBS). Which posttreatment test will the nurse anticipate to be ordered? >Electrolyte panel >Electroencephalogram (EEG) >Lumbar puncture >Urinalysis

Electrolyte panel For the client receiving plasmapheresis for treatment of GBS, the nurse expects that an electrolyte panel will be ordered. Electrolytes will be checked since citrate-induced hypocalcemia is a complication of plasmapheresis. An electroencephalogram evaluates brain waves and is useful in detecting seizure activity. It would not be beneficial in this situation. A lumbar puncture might have been performed as part of the diagnostic process initially but not as part of posttreatment. There is no role for a urinalysis after plasmapheresis.

A client admitted with cerebral edema suddenly begins to have a seizure while the nurse is in the room. What does the nurse do first? >Administer phenytoin (Dilantin). >Draw the client's blood. >Establish an airway. >Start an intravenous (IV) line.

Establish an airway. When a client admitted with cerebral edema begins to have a seizure, the nurse must first establish an airway. The primary goal is to open and maintain an airway and then assess the client for the need of additional support during the seizure. Phenytoin (Dilantin) is administered to prevent the recurrence of seizures, not to treat a seizure already underway. Drawing blood or starting an IV is not the priority in this situation. Remember the ABCs during an emergency situation.

A client with a T6 spinal cord injury who is on the rehabilitation unit suddenly develops facial flushing and reports a severe headache. Blood pressure is elevated, and the heart rate is slow. Which action does the nurse take first? >Check for fecal impaction. >Help the client sit up. >Insert a straight catheter. >Loosen the client's clothing.

Help the client sit up. The nurse's first action for a T6 spinal cord injury client suddenly developing facial flushing and severe headache is to help the client sit up. The client is experiencing autonomic dysreflexia, which can produce severe and rapidly occurring hypertension. Getting the client to sit upright is the easiest and quickest action to take and has the most immediate chance of lowering blood pressure to the brain. Checking for fecal impaction, inserting a straight catheter, and loosening the clothing are important but will not immediately reduce blood pressure.

Which are risk factors for stroke? Select all that apply. >High blood pressure >Previous stroke or transient ischemic attack (TIA) >Smoking >Use of oral contraceptives >Female gender

High blood pressure Previous stroke or transient ischemic attack (TIA) Smoking Use of oral contraceptives Common modifiable risk factors for developing a stroke include smoking and the use of oral contraceptives. Other risk factors include high blood pressure and history of a previous TIA Gender is not a known risk factor for stroke; however, the female client is at risk for delayed recognition of early stroke symptoms.

A client has been admitted with a diagnosis of stroke (brain attack). The nurse suspects that the client has had a right hemisphere stroke because the client exhibits which symptoms? >Aphasia and cautiousness >Impulsiveness and smiling >Inability to discriminate words >Quick to anger and frustration

Impulsiveness and smiling Impulsiveness and smiling are signs and symptoms indicative of a right hemisphere stroke. Aphasia, cautiousness, the inability to discriminate words, quick to anger, and frustration are signs and symptoms indicative of a left hemisphere stroke.

A client has returned to the unit after a thymectomy and is extubated. The client begins to report chest pain. What does the nurse do next? >Calls the Rapid Response Team for immediate intubation >Gives sublingual nitroglycerin (Nitrostat) >Increases the intravenous (IV) rate >Informs the surgeon immediately

Informs the surgeon immediately When a postoperative thymectomy client complains of chest pain, the nurse's next step is to call the surgeon right away. The client's chest pain could be a symptom of a hemothorax or pneumothorax and must be reported to the surgeon immediately. It may be beneficial to notify the Rapid Response Team based on hospital policy; however this would not be the next step. Intubation is not absolutely indicated in this case. The cause of chest pain is unclear but likely related to the thymectomy; therefore, sublingual nitroglycerin would not be beneficial. Increasing the intravenous (IV) rate is not indicated.

A client has been admitted with new-onset status epilepticus. Which seizure precautions does the nurse put in place? Select all that apply. >Bite block at the bedside >Intravenous access (IV) >Continuous sedation >Suction equipment at the bedside >Siderails raised

Intravenous access (IV) Suction equipment at the bedside Siderails raised Seizure precautions the nurse institutes for an admitted client with new-onset status epilepticus include IV access, suctioning equipment at the bedside and raised siderails. IV access is needed to administer medications. Suctioning equipment must be available to suction secretions and facilitate an open airway during a seizure. Raised, padded siderails may be used to protect the client from falling out of bed during a seizure. Bite blocks or padded tongue blades would not be used because the client's jaw may clench, causing teeth to break and possibly obstructing the airway. Continuous sedation is a medical intervention and not a seizure precaution.

The nurse is monitoring a postoperative craniotomy client with increased intracranial pressure (ICP). Which pharmacologic agent does the nurse expect to be requested to maintain the ICP within a specified range? >Dexamethasone (Decadron) >Hydrochlorothiazide (HydroDIURIL) >Mannitol (Osmitrol) >Phenytoin (Dilantin)

Mannitol (Osmitrol) In a postoperative craniotomy client with ICP, the nurse expects Mannitol to be requested to keep the ICP within a certain range. Mannitol is an osmotic diuretic used specifically to treat cerebral edema. Glucocorticoids have no demonstrated benefit in reducing ICP. Hydrochlorothiazide is only a mild diuretic and is not beneficial in maintaining ICP. Dilantin is used to treat seizure activity caused by increased ICP.

The nurse has just received change-of-shift report about a group of clients on the neurosurgical unit. Which client does the nurse attend to first? >Adult postoperative left craniotomy client whose hand grip is weaker on the right >Middle-aged adult client who had a cerebral aneurysm clipping and is becoming increasingly confused >Older adult client who had a carotid endarterectomy and is unable to state the day of the week >Young adult client involved in a motor vehicle crash (MVC) who is yelling obscenities at the nursing staff

Middle-aged adult client who had a cerebral aneurysm clipping and is becoming increasingly confused After a change-of-shift report, the neurosurgical nurse would first attend to the middle-aged client who had a clipping of a cerebral aneurysm and is now becoming increasingly confused. A change in level of consciousness is an early indication that central neurologic function has declined. The primary care provider must be notified immediately. The other clients are not the nurse's first priority. The young adult who is post-MVC does need to be assessed, but the client's behavior does not indicate a decline in neurologic function. The postoperative left craniotomy client and the older adult also need to be assessed, but these clients' neurologic assessment indicates better function.

A client is admitted with a stroke (brain attack). Which tool does the nurse use to facilitate a focused neurologic assessment of the client? >Glasgow Coma Score (GCS) >Intracranial pressure monitor >Mini-Mental State Examination (MMSE; mini-mental status examination) >National Institutes of Health Stroke Scale (NIHSS)

National Institutes of Health Stroke Scale (NIHSS) The nurse uses the NIHSS tool to perform a focused neurologic assessment. Health care providers and nurses at designated stroke centers use a specialized stroke scale such as the NIHSS to assess clients. The Glasgow Coma Score (GCS) provides a non-specific indication of level of consciousness. An intracranial pressure monitor would be requested by the health care specialist if signs and symptoms indicated increased intracranial pressure. The MMSE is used primarily to differentiate among dementia, psychosis, and affective disorders.

Which information is most important for the nurse to communicate to the primary care provider (PCP) about a client who is scheduled for CT angiography? >Allergy to penicillin >History of bacterial meningitis >Poor skin turgor and dry mucous membranes >The client's dose of metformin (Glucophage) held today

Poor skin turgor and dry mucous membranes The most important information for the nurse to communicate to the PCP about a client scheduled for a CT angiography is the client with poor skin turgor and dry mucous membranes. This assessment indicates dehydration which places the client at risk for contrast induced nephropathy. Allergy to penicillin, history of bacterial meningitis, and withheld metformin will need to be reported as part of the client hand-off to Radiology (SAFETY).

A client is admitted with an exacerbation of Guillain-Barré syndrome (GBS), presenting with dyspnea. Which intervention does the nurse perform first? >Calls the Rapid Response Team (RRT) to intubate >Instructs the client on how to cough effectively >Raises the head of the bed to 45 degrees >Suctions the client

Raises the head of the bed to 45 degrees The nurse's first action for a client with an exacerbation of GBS who now has dyspnea is to raise the head of the bed to 45 degrees. The head of the client's bed must be elevated to allow for increased lung expansion. This action helps improve the client's ability to breathe. Calling the RRT for intubation may be necessary if dyspnea is severe or oxygen saturation does not respond to oxygen therapy. Close monitoring of respiratory status is indicated because of the acute stages of GBS. Instructing the client on how to cough effectively is not the priority in this case. The client would be suctioned as needed but cautiously to avoid vagal stimulation.

A client receiving propranolol (Inderal) as a preventative for migraine headaches is experiencing side effects after taking the drug. Which side effect is of greatest concern to the nurse? >Dry mouth >Slow heart rate >Tingling feelings >Warm sensation

Slow heart rate The side effect that is the greatest concern for a client taking propranolol for migraine headaches is a slow heart rate. Beta blockers such as propranolol (Inderal) may be prescribed as a preventive medication for migraines. Propranolol causes blood vessels to relax and improves blood flow although the exact mechanism of action in migraines is unclear. The client would be taught how to monitor his or her heart rate and appropriately report any deviations to the primary care provider. Dry mouth is typically associated with tricyclic antidepressants such as nortriptyline. Skin flushing, tingling feelings, and a warm sensation are common side effects with triptan medications and are not indications to avoid using this group of drugs. Nortriptyline may be used as a preventive medication. Triptans are utilized as abortive medications after a migraine begins.

The nurse is assessing a client with a neurologic condition who is reporting difficulty chewing when eating. The nurse suspects that which cranial nerve has been affected? >Abducens (CN VI) >Facial (CN VII) >Trigeminal (CN V) >Trochlear (CN IV)

Trigeminal (CN V) The nurse suspects that the trigeminal cranial nerve is affected when a client complains of difficulty chewing when eating. The trigeminal nerve affects the muscles of mastication. The abducens nerve affects eye movement via lateral rectus muscles. The facial nerve affects pain and temperature from the ear area, deep sensations in the face, and taste in the anterior two-thirds of the tongue. The trochlear nerve affects eye movement via superior oblique muscles.

The nurse is caring for a client postoperatively after an anterior cervical diskectomy and fusion. Which assessment finding is of greatest concern to the nurse? >Neck pain is at a level 7 on a 0-to-10 scale. >Serosanguineous fluid oozes onto the neck dressing. >The client is reporting difficulty swallowing secretions. >The client has numbness and tingling bilaterally down the arms.

The client is reporting difficulty swallowing secretions. The nursing assessment finding that is the greatest concern for a postoperative anterior cervical diskectomy client is the client reporting difficulty swallowing secretions. This may indicate swelling in the neck and the potential for compromise of the client's airway. Experiencing neck pain and numbness and tingling bilaterally down the arms are expected findings after this surgery. Serosanguineous fluid oozing onto the neck dressing is also a normal finding after this surgery.

Which statement correctly illustrates the commonality between Guillain-Barré syndrome (GBS) and myasthenia gravis (MG)? >The client's respiratory status and muscle function are affected by both diseases. >Both diseases are autoimmune diseases with ocular symptoms. >Both diseases exhibit exacerbations and remissions of their signs and symptoms. >Demyelination of neurons is a cause of both diseases.

The client's respiratory status and muscle function are affected by both diseases. The correct statement about the commonality between GBS and MG is that both diseases affect the respiratory and muscular system. Both GBS and MG affect clients' respiratory status and muscle function. Only MG is an autoimmune disease with ocular symptoms and is characterized by exacerbations and remissions, whereas GBS has three acute stages. GBS causes demyelination of the peripheral neurons.

The nurse is teaching the spouse and client who has had a brain attack about rehabilitation. Which statement by the spouse demonstrates understanding of the nurse's instruction? >"Frequent stimulation will help with the rehabilitation process." >"My spouse will no longer need to take blood pressure medication." >"Rehabilitation and physical therapy are the same thing." >"The rehabilitation therapist will help identify changes needed at home."

The rehabilitation therapist will help identify changes needed at home." Understanding instructions about brain attack is demonstrated by the statement that the rehabilitation therapist will help identify any needed home changes. The rehabilitation therapist and home health professionals assist the client and family in adapting the home environment to the client's needs and assess the client's need for therapy. An appropriate amount of stimulation based on the client's needs will be determined by the therapist and incorporated into a comprehensive plan. Any medication regimen established for the client after the brain attack must be maintained. Rehabilitation is much more comprehensive than physical therapy.

A client in the emergency department (ED) has slurred speech, confusion, and visual problems and has been having intermittent episodes of worsening symptoms. The symptoms have a gradual onset. The client also has a history of hypertension and atherosclerosis. What does the nurse suspect that the client is probably experiencing? >Embolic stroke >Hemorrhagic stroke >Thrombotic stroke >Transient ischemic attack

Thrombotic stroke The client's signs and symptoms fit the description of a thrombotic stroke due to its gradual onset. Signs and symptoms of embolic stroke have a sudden onset, unlike this client's symptoms. Hemorrhagic strokes more frequently present with sudden, severe headache. Intermittent episodes of slurred speech, confusion, and visual problems are transient ischemic attacks, which often are warning signs of an impending ischemic stroke.

The spouse of the client with Alzheimer's disease is listening to the home health nurse explain the client's drug regimen. Which statement by the spouse indicates an understanding of the nurse's instruction? >"Donepezil (Aricept) will treat the symptoms of Alzheimer's disease." >"Memantine (Namenda) is indicated for treatment of early symptoms of Alzheimer's disease. >"Rivastigmine (Exelon) is used to treat depression." >"Sertraline (Zoloft) will treat the symptoms of Alzheimer's disease."

"Donepezil (Aricept) will treat the symptoms of Alzheimer's disease." The comment that shows that the spouse understands the nurse's instructions is that Aricept will treat symptoms of Alzheimer's. Cholinesterase inhibitors (e.g., donepezil) are approved for the symptomatic treatment of Alzheimer's disease. This class of medication delays the destruction of acetylcholine (ACh) by the enzyme cholinesterase. Memantine (Namenda) is indicated for advanced Alzheimer's disease. Memantine blocks excess amounts of glutamate which can damage nerve cells. Rivastigmine (Exelon) is a cholinesterase inhibitor that is used to treat Alzheimer's symptoms. Selective serotonin reuptake inhibitors like sertraline (Zoloft) are antidepressants and may be used in Alzheimer's clients who develop depression.

A client who had a craniotomy is transferred to the intensive care unit from the postanesthesia care unit. Which nursing action is most important when caring for this client? a. Take axillary and oral temperatures. b. Encouraging coughing, but discourage deep breathing. c. Administer a prescribed opioid or sedative at the first sign of irritability. d. Report yellow or bloody drainage on the dressing to the healthcare provider immediately.

Answer D Report yellow or bloody drainage on the dressing to the healthcare provider immediately. Yellow drainage may be cerebral spinal fluid, and bloody drainage is a sign of hemorrhage; either one should be reported to the healthcare provider immediately. Axillary temperatures are influenced by environmental conditions. Temperature evaluation must be accurate; therefore the rectal, not axillary, route is most appropriate. Oral temperatures are contraindicated for clients at risk for seizures. While deep breathing expands the lungs and mobilizes secretions to prevent respiratory complications, coughing can increase intracranial pressure and should be avoided. When necessary, secretions may be removed by suctioning because suctioning is less stressful than coughing and less likely to increase intracranial pressure. Administration of opioids and sedatives hinders accurate neurologic assessment because they depress the central nervous system.

Which is the most effective way for a college student to minimize the risk for bacterial meningitis? >Avoid large crowds. >Get the meningococcal vaccine. >Take a high dose vitamin C daily. >Take prophylactic antibiotics.

Get the meningococcal vaccine. The most effective way for a college student to minimize the risk for bacterial meningitis is to get the meningococcal vaccine. Individuals ages 16-21 years have the highest rates of meningococcal infection and need to be immunized against the virus. Avoiding large crowds is helpful, but is not practical for a college student. Taking a high dose of vitamin C every day does not minimize the risk of bacterial meningitis. However, maintaining a healthy lifestyle, with adequate sleep and nutrition, can improve immunity. Taking prophylactic antibiotics is inappropriate because it leads to antibiotic-resistant strains of microorganisms.

The nurse is performing a rapid neurologic assessment on a trauma client. Which assessment findings are normal? Select all that apply. >Decerebrate posturing >Glasgow Coma Score (GCS) 15 >Lethargy >Minimal response to stimulation >Pupil constriction to light

Glasgow Coma Score (GCS) 15 Minimal response to stimulation Normal rapid neurologic assessment findings include a GCS (Glasgow Coma Score) of 15 and pupil constriction to light. The GCS range is between 3 and 15. Pupil constriction is a function of cranial nerve III. The pupils would be equal in size and round and regular in shape and would react to light and accommodation (PERRLA).Decerebrate or decorticate posturing is not normal, as well as pinpoint or dilated and nonreactive pupils. Both of findings are a late sign of neurologic deterioration. In addition, minimal response to stimulation and increased lethargy are not normal findings.

The nurse is providing medication instructions for a client for whom phenytoin (Dilantin) has been ordered for treatment of epilepsy. The nurse instructs the client to avoid which beverage? >Apple juice >Grape juice >Grapefruit juice >Prune juice

Grapefruit juice The nurse instructs the client taking phenytoin for epilepsy to avoid taking grapefruit juice. Some citrus fruits and juices, like grapefruit juice, can interfere with the metabolism of phenytoin potentially leading to an increased blood level and toxicity. Apple, grape, and prune juices are not contraindicated for a client taking phenytoin (Dilantin).

A client's spouse expresses concern that the client, who has Guillain-Barré syndrome (GBS), is becoming very depressed and will not leave the house. What is the nurse's best response? >"Contact the Guillain-Barré Syndrome Foundation International for resources. Here is their contact information." >"Try inviting several people over so the client won't have to go out." >"Let your spouse stay alone. Your spouse will get used to it." >"This behavior is normal."

"Contact the Guillain-Barré Syndrome Foundation International for resources. Here is their contact information." The nurse's best response to a client's spouse about the client with GBS being depressed is referring the client to the GBS Foundation for resources. The Guillain-Barré Syndrome Foundation International (www.gbs-cidp.org) provides resources and information for clients and their families. The Foundation may be able to help the spouse and family find local support groups to assist the family with the transition. Inviting one close friend over is appropriate, but more than one might overwhelm the client. Telling the spouse to let the client say alone and that the behavior is normal is not helpful and inappropriate. Although depression is expected initially, some action does need to be taken to prevent further deterioration.

The nursing instructor asks a nursing student to compare Bell's palsy and trigeminal neuralgia. Which statement by the nursing student is correct? >"Difficulty chewing may occur in both disorders." >"Both are disorders of the autonomic nervous system." >"Facial twitching occurs in both disorders." >"Both disorders are caused by the herpes simplex virus, which inflames and irritates cranial nerve V."

"Difficulty chewing may occur in both disorders." The correct statement about Bell's palsy and trigeminal neuralgia is that problems with chewing can happen in both disorders. Both Bell's palsy and trigeminal neuralgia can affect cranial nerve V, which affects facial expressions and chewing. Both Bell's Palsy and trigeminal neuralgia are disorders of the cranial nerves. Facial twitching can be a sign of trigeminal neuralgia, whereas Bell's palsy causes a unilateral facial paralysis. Bell's palsy is caused by the herpes simplex virus, unlike trigeminal neuralgia, which is thought to be caused by excessive firing of irritated nerve fibers in the trigeminal nerve.

A client is scheduled for an electroencephalogram (EEG) in the morning. Which instruction does the nurse give the client? >"Do not take any sedatives 12-24 hours before the test." >"Please do not have anything to eat or drink after midnight." >"You may bring some music to listen to for distraction." >"You will need to have someone to drive you home."

"Do not take any sedatives 12-24 hours before the test." Before an EEG, the client needs to be instructed not to use sedatives or stimulants for 12-24 hours prior to the test. A client would not fast prior to an EEG as hypoglycemia may alter results. Testing takes place in a quiet room, so music for distraction is not appropriate. Unless the EEG is for sleep disorder diagnosis, the client will not need to be driven home.

The nurse is developing a teaching plan for a client with a history of low back pain. Which instructions does the nurse plan to include in teaching the client about preventing low back pain and injury? Select all that apply. >"Do not wear high-heeled shoes." >"Keep weight within 50% of ideal body weight." >"Begin a regular exercise program." >"When lifting something, the back should be straight and the knees bent." >"Standing for long periods of time will help to prevent low back pain."

"Do not wear high-heeled shoes." "Begin a regular exercise program." "When lifting something, the back should be straight and the knees bent." The nurse includes the following instructions into the low back pain client's teaching plan: don't wear high-heeled shoes, begin a regular exercise program, and keep the back straight and knees bent when lifting something. Wearing high-heeled shoes can increase back strain. Beginning a regular exercise program will help to promote back strengthening. Keeping the back straight while bending the knees is the proper way to lift objects and will help to prevent back injury. The client needs to avoid standing or sitting for long periods of time because this can cause further strain on the back. Weight needs to be kept within 10% of ideal body weight and not 50%.

The nurse's friend fears that something is wrong with his grandmother, saying that she is becoming extremely forgetful and disoriented and is beginning to wander. What is the nurse's best response? >"Have you taken her for a check-up?" >"She has Alzheimer's disease." >"That is a normal part of aging." >"You should look into respite care."

"Have you taken her for a check-up?" The best response by the nurse to a friend whose grandmother is forgetful and wandering is to ask her friend if he/she has taken the grandmother for a check-up. The grandmother's symptoms could indicate possible Alzheimer's disease or some other physiologic imbalance, and she needs to be assessed further by the primary care provider. The nurse's role is not to diagnose Alzheimer's disease but to advocate for the friend's grandmother to be evaluated. Becoming extremely forgetful, disoriented, and wandering is not normal age related behavior. Respite care is for caregivers, not for clients.

A client newly diagnosed with myasthenia gravis (MG) is being discharged, and the nurse is teaching about proper medication administration. Which statement by the client demonstrates a need for further teaching? >"It is important to post my medicine schedule at home, so my family knows my schedule." >"I can continue to take over-the-counter drugs like before." >"An extra supply of medicine must be kept in my car." >"Wearing a watch with an alarm will remind me to take my medicine."

"I can continue to take over-the-counter drugs like before." Further teaching about medication administration is indicated when the client with MG says that he/she can still take over-the-counter drugs. Clients with MG must not take any over-the-counter medications without checking with their primary health care provider first. The client's medication schedule may be posted in the home for the benefit of family members. An extra supply of medication should be kept in the client's car or workplace to maintain therapeutic levels in case a dose was missed. The client may wear a watch with an alarm as a medication reminder to maintain therapeutic levels.

A client is considering treatments for a malignant brain tumor. Which statement by the client indicates a need for further instruction by the nurse? >"A combination of treatments might be necessary." >"In a craniotomy, holes are cut in the skull to access the tumor." >"I can go home the day of my craniotomy." >"The goal is to decrease tumor size and improve survival time."

"I can go home the day of my craniotomy." The nurse knows that further instruction is needed when a client considering treatment for malignant brain tumor says, "I can go home the day of my craniotomy." Craniotomies are inclient procedures. The client will be admitted to critical care for monitoring after the procedure and may be mechanically ventilated for 24-48 hours postprocedure. Chemotherapy, radiation, and surgery are often used in conjunction with each other to treat malignancies. For a craniotomy, several burr holes are drilled into the skull, and a saw is used to remove a piece of bone (bone flap) to expose the tumor area. The goals of treatment of brain tumor are to decrease tumor size, improve quality of life, and improve survival time.

A client has received preoperative teaching from the nurse for a microdiskectomy. Which statement by the client indicates a correct understanding of the nurse's instruction? >"I can go home the day of the procedure." >"I can go home 48 hours after the procedure." >"I'll have a drain in place after the procedure." >"I'll need to wear special stockings after the procedure."

"I can go home the day of the procedure." The statement that indicates the client correctly understands preoperative teaching of a microdiskectomy is "I can go home the day of the procedure." A microdiskectomy is considered minimally invasive surgery (MIS) and does not typically require an inclient hospital stay. The client who undergoes a minimally invasive surgery does not have to wait 48 hours after the procedure to return home, will not have a drain in place after the procedure, and will not need to wear special stockings after the procedure. These steps are used in the case of traditional open laminectomy, not MIS.

The nurse is teaching a client, newly diagnosed with migraines, about trigger control. Which statement made by the client demonstrates good understanding of the teaching plan? >"I can still eat Chinese food." >"I must not miss meals." >"It is okay to drink a few wine coolers." >"I need to use fake sugar in my coffee."

"I must not miss meals." The client understands the teaching plan about trigger control for migraines when the client states that he/she must not miss meals. Until triggers are identified, a headache diary would be considered. Missing meals is a trigger for many people suffering from migraines. The client must not skip any meals until the triggers are identified. Chinese food frequently contains monosodium glutamate. Monosodium glutamate-containing foods, alcohol, and artificial sweeteners are triggers for many people suffering from migraines and need to be eliminated until the triggers are identified.

A female client with newly diagnosed migraines is being discharged with a prescription for sumatriptan (Imitrex). Which comment by the client indicates an understanding of the nurse's discharge instructions? >"Birth control is not needed while taking sumatriptan." >"I must report any chest pain right away." >"St. John's wort can also be taken to help my symptoms." >"Sumatriptan can be taken as a last resort."

"I must report any chest pain right away." The client comment that shows that she understands the discharge instructions is that any chest pain must be reported right away. Chest pain must be reported immediately with the use of sumatriptan because triptans cause vasoconstriction. Remind the client to use contraception (birth control) while taking the drug because it may not be safe for women who are pregnant. Triptans would not be taken with selective serotonin reuptake inhibitors or St. John's wort, an herb used commonly for depression. Sumatriptan must be taken as soon as migraine symptoms appear.

The nurse is teaching a client and family about home care after a stroke. Which statement made by the client's spouse indicates a need for further teaching? >"I should spend all my time with my husband in case I'm needed." >"My husband may get depressed." >"My husband must take his medicine every day to prevent another stroke." >"The physical therapist will show us how to use the equipment so my husband can climb the stairs and get into and out of bed."

"I should spend all my time with my husband in case I'm needed." Further home care teaching is needed when the stroke client's wife says that "I need to spend all my time with my husband in case I'm needed." Although well intentioned, family members can start to feel socially isolated when caring for a loved one. The family may need to plan for regular respite care in a structured day-care respite program or through relief provided by a friend or neighbor. The life changes associated with stroke often cause a change in the client's self-esteem. The client who has had a stroke needs to maintain a regular medication regimen to help prevent another stroke. If it is determined necessary after a home assessment, the physical and occupational therapist will show the client and family how to use equipment so they are able to mobilize and function in the home setting.

A client with myasthenia gravis (MG) is receiving cholinesterase inhibitor drugs to improve muscle strength. The nurse is educating the family about this therapy. Which statement by a family member indicates a correct understanding of the nurse's instruction? >"I will call 911 if a sudden increase in weakness occurs." >"I will increase the dose if a sudden increase in weakness occurs." >"The medication must be taken with a large meal." >"The medication must be taken on an empty stomach."

"I will call 911 if a sudden increase in weakness occurs." The statement about cholinesterase inhibitors that shows a correct understanding of the nurse's instructions is that the family member will call 911 if there is a sudden increase in weakness. A potential adverse effect of cholinesterase inhibitors is cholinergic crisis. Sudden increases in weakness and the inability to clear secretions, swallow, or breathe adequately indicate that the client is experiencing crisis. The family member must call 911 for emergency assistance. The dose of cholinesterase inhibitors would never be increased without provider supervision. The client needs to eat meals 45-60 minutes after taking cholinesterase inhibitors to avoid aspiration. Cholinesterase inhibitors must be taken with a small amount of food to help alleviate GI side effects.

A client has been diagnosed with Primary Progressive MS (PPMS) and the nurse is providing education at the clinic. What statement by the client indicates the need for more teaching? >"I can alternate wearing my eye patch between eyes for double vision." >"I should keep my home clutter free so I don't fall." >"It's important I work out in the afternoon so my muscles are warmed up." >"I always keep my medications in the same place."

"It's important I work out in the afternoon so my muscles are warmed up." More teaching is needed for the client with PPMS when the client says, "It's important I work out in the afternoon so my muscles are warmed up." Working out in the afternoon will increase body temperature and lead to fatigue. Fatigue is a key feature of MS. Working with a physical therapist to develop an appropriate exercise program tailored to the client's condition will be beneficial. If a client has diplopia, wearing an eye patch and alternating it between eyes every few hours may relieve the symptoms. Keeping the home organized and clutter free will decrease the risk of falls. Keeping medications and other important belongings in the same place and maintaining a routine may help with memory deficits that may occur with MS.

The nurse is providing instructions to a client with a spinal cord injury about caring for the halo device. The nurse plans to include which instructions? >"Avoid using a pillow under the head while sleeping." >"Begin driving 1 week after discharge." >"Keep straws available for drinking fluids." >"Swimming is recommended to keep active."

"Keep straws available for drinking fluids." The instructions the nurse include for a client with a halo device is to keep straws available for drinking fluids. The halo device makes it difficult to bring a cup or a glass to the mouth. The head would be supported with a small pillow when sleeping to prevent unnecessary pressure and discomfort. Driving must be avoided because vision is impaired with the device. Swimming must be avoided to prevent the risk for infection.

A client is eating a soft diet while recovering from a stroke. The client reports food accumulating in the cheek of the affected side. What is the nurse's best response? >"Next time you eat, try lifting your chin when you swallow." >"Let's advance your diet to solid food." >"Let's see if the dietitian can help." >"Let's see if the speech-language pathologist can help."

"Let's see if the speech-language pathologist can help." The nurse's best response about food gathering in the cheek of a stroke client is to see what the speech pathologist says may help. The speech-language pathologist identifies strategies to prevent food from accumulating in the cheek of the affected side of a client recovering from a stroke. The speech pathologist will assist the client with tongue exercises that will help move the food bolus to the unaffected side. Lifting the chin is not an appropriate technique. A solid diet would not necessarily be the best choice. The dietitian will be consulted to evaluate the nutritional status of the client as well as make recommendations regarding the correct diet.

The home health nurse is checking in on a client with dementia and the client's spouse. The spouse confides to the nurse, "I am so tired and worn out." What is the nurse's best response? >"Can't you take care of your spouse?" >"Establishing goals and a daily plan can help." >"Make sure you take some time off and take care of yourself too." >"That's not a very nice thing to say."

"Make sure you take some time off and take care of yourself too." The nurse's best response to the spouse of the client with dementia is to encourage the wife to take some time off to take care of herself. This response is supportive and reminds the spouse that he or she cannot care for the client when exhausted. Questioning the spouse's ability to provide care is not supportive and may offend the spouse. Establishing goals and a daily plan may be helpful to the situation but is not responding to the spouse's need. Reprimanding the spouse does not validate his or her feelings and does not allow the nurse to further explore the statement.

A client with new-onset Bell's palsy is being dismissed from the hospital. Which statement made by the client demonstrates a need for further teaching by the nurse? >"I'll need artificial tears at least four times a day." >"I will eat a soft diet." >"My eye must be taped or patched at bedtime." >"Narcotics will be needed for pain relief."

"Narcotics will be needed for pain relief." Mild analgesics, not narcotics, are used for pain associated with Bell's palsy. Further teaching about Bell's palsy is needed when the client says that narcotics are needed for pain. Artificial tears need to be taken at least 4 times a day and taping the affected eye at night protects the cornea from drying out and potentially ulcerating. Drying out of the eyes occurs because of the eye's inability to close. Mastication is often impaired with Bell's palsy, so soft foods are indicated.

The nurse is teaching a client newly diagnosed with multiple sclerosis (MS). Which statement by the client indicates a correct understanding of the pathophysiology of the disease? >"I will die early." >"I will have gradual deterioration with no healthy times." >"Parts of my nervous system have plaques." >"This was caused by getting too many x-rays as a child."

"Parts of my nervous system have plaques." The statement that demonstrates that the newly diagnosed client with MS correctly understands the pathophysiology of the disease is "parts of my nervous system have plaques." MS is characterized by an inflammatory response that results in diffuse random or patchy areas of plaque in the white matter of the central nervous system. The client with MS has no decrease in life expectancy. Frequent times of remission are common in clients with MS. There is no known cause for MS.

A family member of a client with a recent spinal cord injury asks the nurse, "Can you please tell me what the real prognosis for recovery is? I don't feel like I'm getting a straight answer." What is the nurse's best response? >"Every injury is different, and it is too soon to have any real answers right now." >"Only time will tell." >"The Health Insurance Portability and Accountability Act requires that I obtain the client's permission first." >"Please request a meeting with the health care provider. I will help set that up."

"Please request a meeting with the health care provider. I will help set that up." The nurse's best response to a family member of a client with a recent spinal cord injury is, "Please request a meeting with the primary health care provider. I will set that up." Questions concerning prognosis and potential for recovery would be referred to the primary health care provider. The nurse can help facilitate the meeting however. The timing and extent of recovery are different for each client, but it is not the nurse's role to inform the client and family members of the client's prognosis. Telling the family that "only time will tell" is too vague and minimizes the family's concern. The client was informed of Health Insurance Portability and Accountability Act (HIPAA) rights on admission or when consciousness was established, so permission has already been granted by the client.

A client is being discharged home after surgery for brain cancer. Which statement by the client's spouse indicates a correct understanding of the nurse's discharge teaching? >"I will have to quit my job to care for my spouse." >"Life will be back to normal soon." >"The case manager will provide home care." >"We can find a support group through the local American Cancer Society."

"We can find a support group through the local American Cancer Society." The statement by the spouse of a brain cancer client that shows correct understanding of discharge teaching is when the spouse says, "We can find a support group through the local American Cancer Society." The American Cancer Society is a good community resource for clients with malignant tumors and their families. It is not a requirement that the client's spouse quit his or her job but may need some assistance in home. A diagnosis of brain cancer is life changing and the client and spouse will find a "new normal"; however this will not happen immediately. The case manager helps coordinate care and will be able to locate home care but does not provide that care.

Which client will the neurologic unit charge nurse assign to a registered nurse who has floated from the labor/delivery unit for the shift? >Adult client who has just returned from having a cerebral arteriogram and needs vital sign checks every 15 minutes. >Older adult client who was just admitted with a stroke and needs an admission assessment. >Young adult client who has had a lumbar puncture and reports, "Light hurts my eyes." >Middle-aged client who has a possible brain tumor and has questions about the scheduled magnetic resonance imaging.

Adult client who has just returned from having a cerebral arteriogram and needs vital sign checks every 15 minutes. The charge nurse would assign an RN with experience in labor and delivery to check vital signs and limbs on a client who just returned from a cerebral angiogram. This float nurse would also be able to recognize signs of bleeding. The older adult admitted with a stroke, the young adult post lumbar puncture, and the middle-aged client with a possible brain tumor all require a nurse with more experience with neurologic diagnoses and diagnostic procedures.

The nurse reviews the diagnostic reports of a client and discovers that the client has an injury to cranial nerve VII. What will the nurse observe upon assessment? a. Inhibition of tear production b. Inhibition of peripheral vision c. Impairment of eye movement d. Impairment of pupil constriction

Answer: A Inhibition of tear production Injury to cranial nerve VII mainly leads to inhibition of tear production, a condition called keratoconjunctivitis sicca or dry eye syndrome. Any impairment to cranial nerve II may affect peripheral and central vision. Cranial nerves III, IV and VI affect eye movement. Therefore any injury to these nerves may affect eye movement. The function of cranial nerve III is constriction of the pupil. Any injury to this nerve may lead to impairment of pupil constriction.

Bed rest is prescribed after a client's cerebrovascular accident (CVA, "brain attack") results in right hemiplegia. Which exercises should the nurse incorporate into the client's plan of care 24 hours after the brain attack? a. Passive range of motion exercises b. Active exercises of the extremities c. Light weight-lifting exercises of the right side d. Isotonic exercises that will capitalize on returning muscle function

Answer: A Passive range-of-motion exercises prevent the development of deformities (e.g., contractures) and do not require any energy expenditure by the client. Instituting range-of-motion exercises is an independent nursing function. The client will be unable to perform active exercises and weight-lifting. Isotonic exercises are active movement, which the client is unable to do.

A client is admitted with a closed head injury sustained in a motor vehicle accident (MVA). The nursing assessment indicates increased intracranial pressure (ICP). Which intervention should the nurse perform first? a. Place the head and neck in alignment. b. Administer 1 gram mannitol intravenously (IV) as prescribed. c. Increase the ventilator's respiratory rate to 20 breaths/minute. d. Administer 100 mg of pentobarbital IV as prescribed.

Answer: A Place the head and neck in alignment The nurse should first attempt nursing interventions such as placing the head and neck in alignment (neutral position) to facilitate venous return and thereby decrease ICP. If nursing measures prove ineffective, notify the healthcare provider, who may prescribe mannitol. The nurse would notify the healthcare provider for hyperventilation therapy or for pentobarbital. Hyperventilation is used only when all other interventions have been ineffective in decreasing ICP.

The nurse is caring for a client who had a brain attack (cerebrovascular accident) and who has varying moods. The moods range from anger to depression to concern about the aphasia, hemiparesis, and the gavage feedings. Which behavior best indicates the client's acceptance of physical limitations? a. Performs tube feedings without assistance b. Allows family members to assist with care c. Smiles and becomes more extroverted d. Walks in the hall and sits in the lounge

Answer: A The best indicator of acceptance is when the client begins to participate in self-care (tube feedings). Allowing others to provide care does not indicate acceptance. The nurse cannot assume that physical limitations have been accepted just because a client smiles. Walking in the hall and sitting in the lounge do not indicate acceptance; they may be an attempt to relieve boredom.

After interacting with a client, the nurse believes the client is in the prodromal phase of a migraine. Which statements made by the client led the nurse to reach this conclusion? Select all that apply. a. "I feel drowsy all the time." b. "I feel severe pain over my ear." c. "I feel a throbbing pain in my head." d. "I feel confused at this point in time." e. "I feel weakness in the left side of my body."

Answer: A, D, E The nurse concludes the client is in the prodromal phase by the client's statements of drowsiness, confused state, and weakness on one side of the body. A migraine is a clinical syndrome that is characterized by recurrent episodic attacks of head pain. The first phase of a migraine headache is called the prodromal phase. In the prodromal phase, a variety of neurologic changes are seen. These include drowsiness, acute confusion, vertigo, numbness and tingling of lips or tongue, aphasia, and unilateral weakness. Severe pain over the ear is pain in the templar region and is the second phase of a migraine headache. Throbbing pain in the head occurs in the third phase of a migraine.

A nurse is working with an unlicensed assistive personnel (UAP) in caring for a group of clients. Which statement by the UAP indicates a correct understanding of the UAP's role? a. "I will turn off clients' IVs that have infiltrated." b. "I will take clients' vital signs after their procedures are over." c. "I will use unit written materials to teach clients before surgery." d. "I will help by giving medications to clients who are slow in taking pills."

Answer: B "I will take clients' vital signs after their procedures are over." Monitoring vital signs after procedures is within the scope of a UAP's role. Registered professional nurses or licensed practical nurses, not UAPs, should perform turning off clients' intravenous (IV) infusions that have infiltrated. Using unit written materials to teach clients before surgery should be performed by registered professional nurses or licensed practical nurses, not UAPs. Helping by giving medications to clients who are slow in taking pills should be performed by registered professional nurses or licensed practical nurses, not UAPs.

The nurse is caring for a client with a head injury. The Glasgow Coma Scale score of the client is 9. Which nursing intervention should be performed in the client immediately? a. Intubate the client b. Stabilize cervical spine c. Administer oxygen via a nonrebreather mask d. Control external bleeding with a sterile pressure dressing

Answer: B A client with a head injury may suffer shortness of breath. Therefore the client's cervical spine should be stabilized immediately to maintain the airway. The client should be intubated if the Glasgow Coma Scale score is less than 8. Oxygen should be administered after stabilizing the cervical spine of the client. External bleeding should be controlled after ensuring a patent airway.

When two nurses are getting an older adult out of bed, the client reports feeling light-headed. The nurse identifies that the client's pulse is stable and the client's color has not changed. What should the nurses assist the client to do? a. Slide slowly to the floor to prevent a fall and injury. b. Sit on the edge of the bed while they hold the client upright. c. Bend forward because this will increase blood flow to the brain. d. Lie down quickly so the legs can be raised above the heart level.

Answer: B Sitting allows the nurses to support the client until orthostatic hypotension subsides. The client's stable pulse and color indicate that the situation does not warrant placing the client in the supine position. Sliding slowly to the floor to prevent a fall and injury, bending forward, or rapid movement will permit flexion of the vertebrae, which may traumatize the spinal cord. A light-headed feeling usually is transient until the body adapts to the upright position, so leg elevation is unnecessary.

The nurse observes a window washer fall 25 feet (7.6 m) to the ground, rushes to the scene, and determines that the person is in cardiopulmonary arrest. What should the nurse do first? a. Feel for a pulse b. Begin chest compressions c. Leave to call for assistance d. Perform the abdominal thrust maneuver

Answer: B According to the American Heart Association and Heart and Stroke Foundation of Canada for CPR, the first step is to feel for a pulse after unresponsiveness is established. In this case, the nurse has established that the client has no pulse when cardiopulmonary arrest was determined. Therefore, chest compressions should be initiated immediately. Never leave the client to call for assistance; either call the emergency medical services (EMS) by dialing 911 in the US or 112 in Canada on a cellular phone (and leave the phone on so that EMS can find you) or shout out to others in the area for assistance in seeking EMS. The longer the client goes without circulation, the higher the risk of death, so initiating chest compressions has highest priority when cardiopulmonary arrest has been established. The abdominal thrust (Heimlich) maneuver is used to relieve airway obstruction and is not appropriate in this instance.

How should a nurse prepare a client for cranial surgery? a. Assist the client with securing a wig onto the head. b. Obtain the client's consent to have the head shaved. c. Shampoo the client's hair with a medicated shampoo. d. Tell the client that head shaving is needed before anesthesia.

Answer: B Because of legal and cosmetic concerns, consent for head shaving must be obtained before a client receives anesthesia. Because head shaving is a nursing intervention, the nurse should obtain this consent. The surgeon will obtain the consent for surgery. The client will not be able to wear a wig to surgery as wigs could become a source of infection and should not be worn until healing occurs. Washing the client's hair with a medicated shampoo is unnecessary because the hair will be shaved to help prevent contamination of the surgical site. Shaving of some areas of the head or the entire head is typically performed after the client is anesthetized.

The nurse is conducting a neurologic assessment on a client brought to the emergency room after a motor vehicle accident. While assessing the client's response to pain, the client pulls his arms upward and inward. The nurse recognizes that this response represents an injury to what part of the brain? a. Frontal lobe b. Midbrain c. Pons d. Brainstem

Answer: B Decorticate posturing is a sign of significant deterioration in a client's neurologic status and is manifested by rigid flexing of elbows and wrists. This can represent an injury to the midbrain. Damage to the frontal lobe would affect motor function, problem solving, spontaneity, memory, language, initiation, judgment, impulse control, and social and sexual behavior. The pons (which is part of the brainstem) and brainstem help control breathing and heart rate, vision, hearing, sweating, blood pressure, digestion, alertness, sleep, and sense of balance. Damage to this area would manifest itself as abnormal responses in the above listed areas.

A nurse completes an admission assessment on a client who is diagnosed with myasthenia gravis. Which clinical finding is the nurse most likely to identify? a. Problems with cognition b. Difficulty swallowing saliva c. Intention tremors of the hands d. Nonintention tremors of the extremities

Answer: B Facial muscles innervated by the cranial nerves often are affected; dysphagia, ptosis, and diplopia are present. Myasthenia gravis is a neuromuscular disease with altered neuromuscular junction and receptors, not central nervous system symptoms (problems with cognition). Intention tremors of the hands are associated with multiple sclerosis. Nonintention tremors of the extremities are associated with Parkinson disease.

The family member of a client with newly diagnosed Guillain-Barré syndrome comes out to the nurse's station and informs the nurse that the client is having difficulty breathing. What is the first action the nurse should do? a. Notify the healthcare provider. b. Go with the family member to assess the client. c. Send the nursing assistive personnel to take vital signs. d. Assure the family member this is a normal response for this disease.

Answer: B Go with the family member to assess the client. The initial response for the nurse is to assess the client to ensure a patent airway. Guillain-Barré syndrome will exhibit ascending paralysis and can impede respiratory function. The healthcare provider will be notified after the nurse has assessed the client. The nurse needs to personally assess the client since this is a change in condition; the nurse should not send the nursing assistive personnel to assess the client. This is not a normal response to this disease, so it is not correct to assure the family member of this.

A client has expressive aphasia. The client's family members ask how they can help the client regain as much speech function as possible. Which information should the nurse share with the family? a. Speak louder than usual during visits while looking directly at the client. b. Encourage the client to speak while allowing time to respond. c. Give positive reinforcement for correct communication. d. Tell the client to use the correct words when speaking.

Answer: B In addition to the extent of injury, a factor in relearning speech is the client's motivation and effort. The more the client attempts to talk, the more likely speech will progress to its optimum level; relearning is a slow process. Clients with aphasia are not deaf. Although the nurse should instruct the family to approve and support the client's efforts to communicate, this support should be for the effort, not for correct communication. Telling the client to use the correct words when speaking will create frustration and may anger the client.

A client experiences a cerebral vascular accident (CVA) and is admitted to the hospital in a coma. What is the priority nursing care for this client? a. Monitor vital signs b. Maintain an open airway c. Maintain fluid and electrolytes d. Monitor pupil response and equality

Answer: B Maintain an open airway A patent airway is the priority because the airway may become occluded by the tongue in an unconscious client. Monitoring vital signs is not the priority, although it is an important nursing function. Monitoring pupil response and equality and maintaining fluid and electrolytes are not the priority, although they are important nursing functions.

A nurse enters the room of a client with myasthenia gravis and identifies that the client is experiencing increased dysphagia. What should the nurse do first? a. Administer oxygen b. Raise the head of the bed c. Perform tracheal suctioning d. Call the healthcare provider

Answer: B Raising the head of the bed allows gravity to assist in the swallowing of saliva, thus decreasing the risk for aspiration. Oxygen will not assist in the management of dysphagia or the prevention of aspiration. Performing tracheal suctioning may become necessary if the upright position does not allow the client to manage secretions. Alerting the healthcare provider to the problem is necessary, but only after client safety is ensured.

After a cervical neck injury, a client is placed in a halo fixation device with a body cast. Which statement indicates the client's concern about body image has been resolved successfully? a. "I hate having everyone else do things for me." b. "I've gotten used to the brace. I may even miss it when it's gone." c. "I've been keeping my daily calories low in an attempt to lose weight." d. "I can't get to sleep. However, I make up for it in the morning by sleeping later."

Answer: B The client is demonstrating acceptance and is looking toward the future with the response "I've gotten used to but the brace. I may even miss it when it's gone." The response "I hate having everyone else do things for me" relates to low self-esteem, not body image disturbance. The response "I've been keeping my daily calories low in an attempt to lose weight" may indicate that the client may not accept the present body weight. Although the response "I can't get to sleep. However, I make up for it in the morning by sleeping later" may indicate adaptability; it is not related to body image.

While caring for a client who sustained a severe head injury in a motor vehicle accident, the nurse observes that the client is constantly passing urine and is dehydrated. What does the nurse suspect as the cause for the client's condition? a. Decreased secretion of aldosterone b. Decreased secretion of antidiuretic hormone c. Decreased secretion of parathyroid hormone d. Decreased secretion of atrial natriuretic peptide

Answer: B The client sustained a head injury in the accident; therefore the nurse suspects that the cause of constant water loss through urine could be because of decreased antidiuretic hormone. Diabetes insipidus is a complication of traumatic brain injury where the posterior pituitary does not secrete antidiuretic hormone. In the absence of antidiuretic hormone, water is not reabsorbed from the tubules in the nephron and, therefore, gets eliminated as urine. Aldosterone is secreted by the adrenal cortex and mainly controls sodium-potassium levels. Parathyroid hormone helps regulate serum calcium levels in the body and is secreted by the parathyroid glands located in the neck. Atrial natriuretic peptide is secreted by the myocyte cells in the right atrium and work in opposition to aldosterone, causing increased urine output.

A man walks into the emergency room (ER) with sunglasses on and tells the nurse that he fell off a ladder and hit his head and was unconscious for a few minutes. What is the most appropriate next question the nurse should ask the client? a. "Did you pass out?" b. "Can you take off your sunglasses?" c. "Are you injured anywhere else?" d. "How many feet did you fall?"

Answer: B The nurse cannot quickly assess the client for raccoon eyes unless the sunglasses are removed. Raccoon eyes is periorbital ecchymosis around the eyes. If bilateral, it is highly suggestive of basilar skull fracture. It is caused by rupture of the meninges causing the venous sinuses to bleed into the arachnoid villi and cranial sinuses, resulting in pooling of blood around the eyes. It most often is associated with fractures of the anterior cranial fossa and requires immediate attention. It is also important to assess for any loss of consciousness, other injuries, and the height of the fall. However, visually assessing the client comes first.

A nurse is teaching a client with multiple sclerosis about the disease. Which statement by the client indicates to the nurse that further teaching is needed? a. "I avoid use of a straw to drink liquids." b. "I will take a hot bath to relax my muscles." c. "I will plan to use an incontinence pad when I go out." d. "I may be having a rough time now, but I hope tomorrow will be better."

Answer: B The nurse needs to address the hot baths to correct this misconception. Hot baths tend to increase symptoms and may result in burns because of decreased sensation. All the rest are correct and do not require teaching. Using a straw gives the client less control of liquid intake, which may lead to aspiration. Although a bladder regimen to maintain control is preferable, the use of pads can avoid embarrassment. The disease does have periods of remission and exacerbation.

A client with a traumatic brain injury is demonstrating signs of increasing intracranial pressure, which may exert pressure on the medulla. What should the nurse assess to determine involvement of the medulla? Select all that apply. a. Taster b. Breathing c. Heart rate d. Fluid balance e. Voluntary movement

Answer: B, C The medulla, part of the brainstem just above the foramen magnum, is concerned with vital functions such as breathing. The medulla is concerned with vital functions such as heart rate. The opercular-insular area of the parietal cerebral lobe is concerned with taste sensations. The medulla is not concerned with fluid balance. Osmoreceptors of the hypothalamus cause increased or decreased antidiuretic hormone (ADH) secretion depending on serum osmolarity. Voluntary movements are mediated through the somatomotor area of the cerebral cortex.

Where are the central thermoreceptors located in the human body? Select all that apply. a. Skin b. Spinal cord c. Hypothalamus d. Throughout the body e. Abdominal organs

Answer: B, C, D Central thermoreceptors in the body provide skin and core temperature information to the hypothalamus and are located in the spinal cord, hypothalamus, and abdominal organs. Central thermoreceptors are not present in the skin; peripheral thermoreceptors are present in the skin. Central thermoreceptors are not present throughout the body; multiple thermoreceptors are present throughout the body.

What should a nurse assess after applying a body jacket brace to a client with severe spine injuries following a car accident? Select all that apply. a. Pin sites b. Development of cast syndrome c. Signs of compartment syndrome d. Abdomen for decreased bowel sounds e. Areas of pressure over the bony prominences

Answer: B, D, E A client with a severe spine injury due to an accident would benefit from application of a body jacket brace, which immobilizes and supports the thoracic and lumbar spine. After application of the brace, the nurse should assess the client for the development of cast syndrome. This condition occurs when a brace is tightly applied, compressing the superior mesenteric artery against the duodenum. A window in the brace may be left over the umbilicus. The nurse should monitor the reduction in bowel sounds to prevent abdominal pressure and pain. The nurse should assess the areas of pressure over the bony prominences such as the iliac crest and then adjust or remove the brace based upon any complications. A client with an external fixator will need pin sites assessed. A client with a lower extremity cast must regularly be assessed for signs of compartment syndrome and increased pressure at the heel, anterior tibia, head of the fibula, and malleoli.

Postoperatively, a client complains about a variety of minor environmental factors, frequently changes positions, and avoids eye contact. The nurse responds to these observations by stating, "Let me get you some cold water and your pain pill, and you'll be much better." What does the nurse's response demonstrate? a. An empathic recognition of anxiety b. Addressing of the client's needs c. An inappropriate interpretation of the assessment findings d. Advanced problem solving and critical thinking skills

Answer: C An inappropriate interpretation of the assessment findings The nurse did not clarify whether the client was in pain; also the nurse's statement offers false reassurance. The nurse's response denies the client's anxiety; it identifies pain as the problem. The nurse's response denies the client's needs; the client needs to discuss concerns and feelings. The nurse inappropriately identified the problem and did not clarify the client's needs.

When caring for a client who has hyponatremia, the nurse would monitor for which symptom? a. Increased urine output b. Deep rapid respirations c. Change in level of consciousness d. Distended neck veins

Answer: C Change in level of consciousness A normal sodium level is between 135 and 145 mEq/L of sodium. As sodium levels drop below 140 mEq/L, symptoms reflect cellular over-hydration which results from water movement from the relatively hypotonic serum into cells. Symptoms affect primarily the central nervous system (CNS) and musculoskeletal systems. CNS effects range from headache, fatigue and anorexia to lethargy, confusion, disorientation, agitation, vomiting, seizures, and coma. Musculoskeletal symptoms may include cramps and weakness. Vital signs will reflect an increased, weak, thready pulse, shallow respirations, and a low urine output.

Which autoimmune disease affects the central nervous system? a. Uveitis b. Celiac disease c. Multiple sclerosis d. Goodpasture syndrome

Answer: C Multiple sclerosis is an autoimmune disease that affects the central nervous system. Uveitis is an autoimmune disease that affects the eyes. Celiac disease is an autoimmune disease that affects the gastrointestinal System. Goodpasture syndrome is an autoimmune disease that affects the kidneys.

A nurse gave a client naloxone. To evaluate the effectiveness of the medication, what should the nurse assess for? a. Change in level of consciousness b. Increased pain c. Increased respiration d. Decreased heart rate

Answer: C Naloxone is given for decreased respirations caused by opioid overdose[1][2]. The amount given is determined by the respiratory status, not the level of consciousness. Undesirable side effects of naloxone are pain and rapid heart rate with dysrhythmias.

A client experiences expressive aphasia as a result of a brain attack (cerebrovascular accident, CVA). The client's spouse asks whether the client's speech will ever return. What is the best response by the nurse? a. "It should return in several months." b. "You will have to ask the primary healthcare provider." c. "It is hard to say how much improvement will occur." d. "Unfortunately, your spouse will no longer be able to speak."

Answer: C Recovery from aphasia is a continuous process; the amount of recovery cannot be predicted. The response "It should return in several months" gives false reassurance; it may take a year or longer or may never return. The response "You will have to ask the primary healthcare provider" abdicates the nurse's responsibility; the healthcare provider cannot predict return of function. Speech return is a continuous process; it may take a year or longer or may never return.

A nurse assesses a client for increasing intracranial pressure by monitoring the pulse pressure. What is the pulse pressure? a. Force exerted against an arterial wall b. Gap between the apical and radial rates c. Difference between systolic and diastolic readings d. Quality of ventricular contraction in relation to cardiac output

Answer: C The pulse pressure is obtained by subtracting the diastolic blood pressure reading from the systolic blood pressure reading; pulse pressure widens as intracranial pressure increases. The force exerted against an arterial wall is reflected in blood pressure readings and indicates cardiovascular function. The gap between the apical and radial rates is the definition of a pulse deficit. The quality of ventricular contraction in relation to cardiac output is determined by various diagnostic techniques used in cardiology; this is the role of the primary healthcare provider, not the nurse.

On the first postoperative evening after a lumbar laminectomy, a client states, "My feet are as numb as they were before the operation." Which is the nurse's best response? a. "Let me elevate your feet so the numbness will decrease more quickly." b. "That's important to know. I will inform your healthcare provider about the numbness." c. "Continue to let me know how you feel. It often takes time before this feeling subsides." d. "There is no cause for concern because the numbness will disappear as soon as the anesthesia wears off."

Answer: C The response "Continue to let me know how you feel. It often takes time before this feeling subsides" offers the realistic assurance that nothing is wrong and encourages the client to relate information to the nurse. The response "Let me elevate your feet so the numbness will decrease more quickly" will not decrease the numbness; nerve root irritation will lessen only with time. The response "That's important to know. I will inform your healthcare provider about the numbness" tells the client that there is a problem when, in reality, there is no reason to call the healthcare provider. The response "There is no cause for concern because the numbness will disappear as soon as the anesthesia wears off" provides false reassurance; nerve root numbness lessens only with time.

A client newly diagnosed with multiple sclerosis asks the nurse if it will be painful. Which response should the nurse give the client first? a. "Tell me more about your fears regarding pain." b. "Medications will be prescribed to help control pain." c. "Pain is a common symptom of this condition." d. "Let's list your questions for the healthcare provider."

Answer: C The response "Pain is a common symptom of this condition" is a truthful answer for the client. Reassuring the client that "medications will be prescribed to help control pain" when the client experiences it is the next helpful response from the nurse. After being truthful about pain and reassuring the client about its medical management, asking the client to "tell more about...fears regarding pain" opens the conversation to discuss it and offers an opportunity for emotional release, which can decrease anxiety. The response "Let's list your questions for the healthcare provider" is a helpful final conversation during this encounter because it teaches the client how to make the most of their visit with the healthcare provider.

nurse should plan to maintain a client who has experienced a subarachnoid hemorrhage in what position? a. Supine b. On the unaffected side c. In bed with the head of the bed elevated d. With sandbags on either side of the head

Answer: C With the head of the bed elevated, the force of gravity helps prevent additional intracranial pressure which will intensify the ischemic manifestations of hemorrhage. The supine position will not facilitate drainage of cerebral fluid; this position promotes accumulation of fluid, which increases intracranial pressure. Lying on the unaffected side will not facilitate drainage of cerebral fluid; this position promotes accumulation of fluid, which increases intracranial pressure. Vomiting can occur with increased intracranial pressure, and placing sandbags to immobilize the head can result in aspiration.

The nurse teaches a nursing student about the discharge instructions to be given to a post-operative client. Which statement made by the nursing student indicates the nurse needs to intervene? a. "I should teach the client about using topical antibiotics." b. "I should teach the client about how to change wound dressings." c. "I should instruct the client about signs and symptoms of an infection." d. "I should instruct the client that the non-oozing wound should be cleaned with saline solution."

Answer: D "I should instruct the client that the non-oozing wound should be cleaned with saline solution." The nurse should intervene to instruct the nursing student to teach the client to clean his or her non-oozing wounds with normal soap and plain water. All the other statements are correct and do not require the nurse to intervene. The nursing student should teach the client about the use of topical antibiotics and how to change wound dressings. The nursing student should instruct the client about the signs and symptoms of infection.

In caring for the client with burr holes for a subdural hematoma postoperatively on day 2, the nurse notes the client has an increased temperature to 101.3 F° (38.5° C). What does the nurse understand about this reaction? a. This is a normal assessment for the client with a subdural hematoma. b. This is a normal reaction day 2 postoperatively, and the nurse will administer acetaminophen as prescribed by the healthcare provider. c. Because the client has burr holes, this is not an accurate measurement. d. The client is exhibiting signs of an infection, and the healthcare provider needs to be notified.

Answer: D Any client with a temperature day 2 postoperatively could be exhibiting signs and symptoms of an infection. The nurse should notify the healthcare provider and continue assessment of the client for signs and symptoms of infection. An increased temperature 2 days postoperatively is not normal for any client. The burr holes have nothing to do with whether or not the temperature is okay.

The bed alarm is ringing because an older adult client is attempting to get out of bed. A nurse enters the room and finds the client agitated and confused. The family member is upset and states, "He has never been like this. I don't know what to do." After getting the client back into bed, which nursing action is most appropriate? a. Asking the family member to step out of the room so the client can rest b. Placing a vest restraint on the client to prevent the client from falling out of bed c. Explaining to the family that it is common for older clients to get confused while in the hospital d. Requesting the nursing assistant to stay with the client while the nurse calls the primary healthcare provider

Answer: D Because this is new for the client, the nurse should notify the primary healthcare provider. The client should be monitored continually for a while to prevent falling or injuring himself. This is an appropriate task to delegate to a nursing assistant. Since this is new for the client, reassuring the family that older adult clients often get confused in the hospital is not helpful. Evidence-based practice has shown that having a family member with the client is helpful. Therefore, the family member should be encouraged to stay with the client. Placing a restraint on the client should be done as a last resort and not instituted without a primary healthcare provider's prescription.

A client falls from a two-story building and is taken to the hospital unconscious. Which finding identified during the initial nursing assessment should be of most concern to the nurse? a. Glasgow Coma Scale (GCS) score of 8 b. Bleeding from the ears c. Pupils reactive to light d. Depressed fontanel

Answer: D Bleeding from the ears occurs with basal skull fractures; this assessment assists with diagnosing the location of the injury. A Glasgow Coma Scale of 8 indicates the client is unconscious, which is already specified in the scenario; therefore, it is not as concerning as the bleeding from the ears. A reactive pupil is a positive response; pupils should react to light. A depressed fontanel occurs in an infant in the presence of dehydration.

A client reports buzzing in the ear for the past 5 days and a decreased ability to hear sounds. After interacting with the client, the nurse finds that the client is receiving long-term aminoglycoside therapy. Which cranial nerve should the nurse suspect to be affected? a. CN III b. CN V c. CN VII d. CN VIII

Answer: D CNVII CN VIII is the vestibulocochlear nerve, a part of central auditory system. Medications such as aminoglycosides are ototoxic and can damage CN VIII and cause hearing loss, tinnitus (buzzing in ears), and vertigo. CN III is the oculomotor nerve that innervates the iris sphincter muscle. This muscle helps in pupil constriction. CN V is the trigeminal nerve that innervates the iris dilator muscle involved in the dilation of the pupil. CN VII is the facial nerve, which innervates the muscles that help open and close the eyelids.

A client with quadriplegia is placed on a tilt table daily. The client asks why the angle of the head of the table is gradually increased. How should the nurse respond? a. It facilitates turning. b. This prevents pressure ulcers. c. It promotes hyperextension of the spine. d. This limits loss of calcium from the bones.

Answer: D During prolonged inactivity, bone resorption proceeds faster than bone formation, and lack of therapeutic weight bearing on bone results in demineralization. A tilt table provides gradual progressive weight bearing, which counters these effects. Lateral turning is possible and necessary if a client is immobile, but a tilt table does not make this possible. The tilt table is used for scheduled periods in physical therapy. The nursing care required to prevent pressure ulcers must be consistently and frequently performed throughout the day and night. The tilt table does not cause hyperextension of the spine; the spine remains in functional body alignment.

A client has a tonic-clonic seizure at work and is admitted to the emergency department. Which question is most useful when planning nursing care related to the client's seizure? a. "Is your job demanding or stressful most of the time?" b. "Do you participate in any strenuous sports activities on a regular basis?" c. "Does anyone in your family have a history of central nervous system problems?" d. "Were you aware of anything different or unusual just before your seizure began?"

Answer: D Identification of a sensation that occurs before each seizure (aura) is helpful in identifying the cause of the seizure and planning how to identify and avoid a future seizure. Although the response "Is your job demanding or stressful most of the time?" may provide some information, it is not the most inclusive question the nurse can ask; also, it limits the client's reply. Although the response "Do you participate in any strenuous sports activities on a regular basis?" may provide some information, it is not the most inclusive question the nurse can ask; also, it limits the client's reply. Although the response "Does anyone in your family have a history of central nervous system problems?" may provide some information, it is not the most inclusive question the nurse can ask; also, it limits the client's reply.

Initially after a stroke, a client's pupils are equal and reactive to light. Later, the nurse assesses that the right pupil is reacting more slowly than the left and that the systolic blood pressure is beginning to rise. What complication should the nurse consider that the client is developing? a. Spinal shock b. Hypovolemic shock c. Transtentorial herniation d. Increasing intracranial pressure

Answer: D Increased intracranial pressure compresses vital brain tissue; this is manifested by a sluggish pupillary response and an increased systolic blood pressure. Spinal shock is manifested by decreased systolic blood pressure, with no pupillary changes. Hypovolemic shock is indicated by decreased systolic pressure and tachycardia, with no changes in pupillary reaction. Transtentorial herniation is manifested by dilated pupils and severe posturing.

A client who was a passenger in an automobile collision is admitted to the emergency department with rhinorrhea and bleeding from the ear. The healthcare provider determines that the client has a basilar head injury. What should the nurse anticipate is the initial focus of care for this client? a. Physical therapy b. Psychosocial support c. Nutritional management d. Antimicrobial administration

Answer: D Preventing infection through the use of prophylactic antibiotics is the priority. Tearing the meninges may have introduced infectious organisms. Physical therapy is premature; physical therapy begun too early can increase intracranial pressure. Although psychosocial support is important, it is not the priority. Nutrition is not the priority at this time.

The spouse of a client who had a cerebrovascular accident (also known as a "brain attack") seems unable to accept the concept that the client must be encouraged to participate in self-care. What is the best response by the nurse? a. Tell the spouse to let the client do things independently. b. Allow the spouse to assume total responsibility for the client's care. c. Explain that the nursing staff has full responsibility for the client's activities. d. Ask the spouse for assistance in planning those activities most helpful to the client.

Answer: D To foster communication and cooperation, family members should be involved in planning and implementing care. Telling the spouse to let the client do things independently does not focus on feelings or needs. The spouse may promote dependency in the client to satisfy a need to control. Although the nursing staff does have full responsibility for the client's activities, the family should be involved.

A client is diagnosed as having a right-sided brain attack (cerebrovascular accident) and is admitted to the hospital. When preparing to care for this client, which intervention should the nurse perform? a. Apply elastic stockings to prevent flaccid leg muscles b. Use a bed cradle to prevent dorsiflexion of the feet c. Implement passive range-of-motion exercises to prevent muscle atrophy d. Use a hand roll while supporting the left upper extremity on a pillow to prevent contractures

Answer: D Using a hand roll while supporting the left upper extremity on a pillow to prevent contractures will maintain the affected left arm in functional alignment; the left side of the body will be affected with a right-sided brain attack. Elastic stockings promote venous return rather than prevent flaccid muscles; also, these require a prescription. Plantar flexion (foot drop), not dorsiflexion, may occur with a brain attack; high-top sneakers or splints, not a bed cradle, more appropriately prevent plantar flexion contractures. Passive ROM exercises prevent contractures rather than muscle atrophy; the institution of ROM exercises should be discussed with the healthcare provider because activity during the acute phase can increase intracranial pressure and should be avoided.

A client returns to the neurosurgical floor after undergoing an anterior cervical diskectomy and fusion (ACDF). What is the nurse's first action? >Administer pain medication. >Assess airway and breathing. >Assist with ambulation. >Check the client's ability to void.

Assess airway and breathing. The nurse's first action when a client returns to the neurosurgical floor after having an anterior cervical discectomy is to assess the airway and breathing. Assessment in the immediate postoperative period after an ACDF is maintaining an airway and ensuring that the client has no problem with breathing. Administration of pain medication, ambulation, and assessing the client's ability to void are important but are not the highest priority.

A client hospitalized for hypertension presses the call light and reports "feeling funny." When the nurse gets to the room, the client is slurring words and has right-sided weakness. What does the nurse do first? >Assesses airway, breathing, and circulation >Calls the provider >Performs a neurologic check >Assists the client to a sitting position

Assesses airway, breathing, and circulation When a client reports "feeling funny" and then starts slurring words and has right-sided weakness, the nurse must first assess for airway, breathing, and circulation. The priority is assessment of the "ABCs"—airway, breathing, and circulation. Calling the Rapid Response Team, not the provider, after assessing ABCs would be appropriate. The first 10 minutes after onset of symptoms is crucial. A neurologic check will be performed rapidly but is not the top priority. The client would be placed in bed, easily accessible for healthcare providers to assess and begin treatment. This does not need to be a seated position.

A client is admitted with bacterial meningitis. Which nursing intervention is the highest priority for this client? >Assessing neurologic status at least every 2-4 hours >Decreasing environmental stimuli >Managing pain through drug and nondrug methods >Strict monitoring of hourly intake and output

Assessing neurologic status at least every 2-4 hours The highest priority nursing intervention for the newly admitted client with bacterial meningitis is to accurately monitor and record the client's neurologic status every 2-4 hours. The neurologic status, vital signs, and vascular status must be assessed at least every 4 hours or more often, if clinically indicated, to rapidly determine any deterioration in status. Decreasing environmental stimuli is helpful for the client with bacterial meningitis but is not the highest priority. Clients with bacterial meningitis report severe headaches requiring pain management which may be accomplished through both pharmacologic and nonpharmacologic methods. Assessing fluid balance while preventing overload is not the highest priority however intake and output must be monitored.

The nurse is assessing a client with a traumatic brain injury after a skateboarding accident. Which sign/symptom is the nurse most concerned about? >Amnesia >Asymmetric pupils >Headache >Head laceration

Asymmetric pupils The nurse is most concerned about asymmetric pupils in the client with traumatic brain injury. Asymmetric (uneven) pupils are treated as herniation of the brain from increased intracranial pressure (ICP) until proven otherwise. The nurse must report and document any changes in pupil size, shape, and reactivity to the primary health care provider immediately. Amnesia, a headache and a head laceration, can be signs of mild traumatic brain injuries and need to be investigated more thoroughly.

Which task does the nurse plan to delegate to the unlicensed assistive personnel (UAP) caring for a group of clients in the neurosurgical unit? >Assist the health care provider in performing a lumbar puncture on a confused client >Attend to the care needs of a client who has had a transcranial Doppler study >Educate a client about what to expect during an electroencephalogram (EEG) >Prepare a client who is going to radiology for a cerebral arteriogram

Attend to the care needs of a client who has had a transcranial Doppler study The nurse delegates the UAP to care for the client who has had a transcranial Doppler study. Since transcranial Doppler studies are noninvasive and do not require any post procedure monitoring or care the UAP can safely attend to this client. Assisting the primary care provider in performing a lumbar puncture and preparing a client for a cerebral arteriogram require assessments and interventions that would be done by licensed nursing staff. Client teaching would also be provided by licensed nursing staff.

The nurse prepares to assess a client with diabetes mellitus for sensory loss. Which equipment is the best choice for the nurse use to perform this assessment? >Cotton-tipped applicator >Glucometer >Hammer >Safety pin

Cotton-tipped applicator A cotton-tipped applicator is the nurse's best choice to assess sensory loss on a client with diabetes mellitus. Sensory loss is assessed with any sharp or dull object, such as a cotton-tipped applicator. The client indicates whether the touch is sharp or dull. The soft and hard ends of the applicator would be interchanged at random so that the client does not anticipate the next type of sensation. A glucometer tests blood sugar. A hammer tests tendon reflexes. Although a safety pin could be used to test for sensory loss, a cotton-tipped applicator is safer in the event the client is taking anticoagulants.

The nurse is monitoring a client after supratentorial surgery. Which sign/symptom does the nurse report immediately to the provider? >Periorbital edema >Bilateral ecchymoses of both eyes >Moderate amount of serosanguineous drainage on the head dressing >Decorticate positioning

Decorticate positioning In a postoperative supratentorial client, the nurse must immediately report decorticate positioning to the provider. The major complications of supratentorial surgery are increased intracranial pressure from cerebral edema or hydrocephalus and hemorrhage. Decorticate positioning indicates damage to the pathway between the brain and the spinal cord. Periorbital edema and a small-to-moderate amount of serosanguineous drainage are expected after a craniotomy. Ecchymoses in the facial region, especially around the eyes, are expected after a craniotomy.

The nurse admits a client with suspected Eaton-Lambert syndrome. The nurse anticipates that the primary health care provider (PHCP) will request which test to confirm the diagnosis? >Doppler study >Electromyography (EMG) >Magnetic resonance imaging (MRI) >Tensilon test

Electromyography (EMG) The nurse expects the PHCP to request an electromyography for the client suspected of having Eaton-Lambert syndrome. EMG is used to confirm the diagnosis of Eaton-Lambert syndrome, which is a form of myasthenia gravis (MG) that is often seen with small cell carcinoma of the lung. Doppler study is used frequently in the diagnosis of vascular disorders. MRI is not used to confirm the diagnosis of Eaton-Lambert syndrome. The Tensilon test is used as a diagnostic test in MG, but it is not used to confirm the diagnosis of Eaton-Lambert syndrome even though it is a form of MG.

A client is being discharged home after treatment for a brain attack. What is the mnemonic that the nurse can teach the family and client to help recognize and act on another stroke? >A-V-P-U >F-A-S-T >K-I-N-D >O-P-Q-R-S-T

F-A-S-T The mnemonic F-A-S-T is utilized to teach the client, family, and community how to recognize and respond to a stroke. The purpose is to observe the Face, Arms, Speech, and then Time of onset and knowing it's Time to call 9-1-1.A-V-P-U is the mnemonic for level of awareness (alert, verbal, painful, and unresponsive). K-I-N-D is a mnemonic for treatment of hyperkalemia (kayexalate, insulin, NaHCO3, diuretics). O-P-Q-R-S-T is a mnemonic for assessing pain (onset, provokes, quality, radiates, severity, time).

A client on the neurosurgical floor who had a lumbar laminectomy is confused, agitated, and complaining of difficulty breathing. The client is normally alert and oriented. The nurse notices a pinpoint rash over the client's chest. What condition is the nurse concerned has occurred? >Autonomic dysreflexia >CSF leak >Fat embolism syndrome >Paralytic ileus

Fat embolism syndrome The nurse is concerned that fat embolism syndrome has occurred. Fat embolism syndrome (FES) is characterized by chest pain, dyspnea, anxiety, and mental status changes. Petechiae may develop around the neck, over the upper chest, buccal mucosa, and conjunctiva. This is an emergency. The nurse must notify the primary health care provider immediately. Autonomic dysreflexia is not associated with lumbar laminectomies. It is seen in spinal cord injuries. A cerebrospinal fluid (CSF) leak is a concern with laminectomy but would not present with these symptoms. Paralytic ileus may occur but is associated with abdominal pain and distention.

A client has had a traumatic brain injury and is mechanically ventilated. Which technique does the nurse use to prevent increasing intracranial pressure (ICP)? >Assessing for Grey Turner's sign >Maintaining neutral head position >Placing the client in the Trendelenburg position >Suctioning the client frequently

Maintaining neutral head position To prevent ICP in a client with traumatic brain injury who is being mechanically ventilated, the nurse needs to maintain the patent's head in a neutral position. Maintaining the head in neutral alignments prevents obstruction of blood flow and is an important component of ICP. Grey Turner's sign is a bluish gray discoloration in the flank region caused by retroperitoneal hemorrhage. The head of the bed needs to be at 30 degrees. The Trendelenburg position will cause the client's ICP to increase. Although some suctioning is necessary, frequent suctioning would be avoided because it increases ICP.

A client has Parkinson's disease (PD). Which nursing intervention best protects the client from injury? >Discouraging the client from activity >Encouraging the client to watch the feet when walking >Monitoring the client's sleep patterns >Suggesting that the client obtain assistance in performing activities of daily living (ADLs)

Monitoring the client's sleep patterns The nursing intervention that best protects the PD client from injury is to monitor the client's sleep patterns. Clients with PD tend to not sleep well at night because of drug therapy and the disease itself. Some clients nap for short periods during the day and may not be aware that they have done so. This sleep misperception could put the client at risk for injury (e.g., falling asleep while driving).Active and passive range-of-motion exercises, muscle stretching, and activity are important to keep the client with PD mobile and flexible. The client with PD needs to avoid watching his or her feet when walking to prevent falls and would be encouraged to participate as much as possible in self-management, including ADLs. Occupational and physical therapists can provide training in ADLs and the use of adaptive devices, as needed, to facilitate independence.

A client arrives in the emergency department with new-onset ptosis, diplopia, and dysphagia. The nurse anticipates that the client will be tested for which neurologic disease? >Bell's palsy >Guillain-Barré syndrome (GBS) >Myasthenia gravis (MG) >Trigeminal neuralgia

Myasthenia gravis (MG) The nurse expects the client with these signs/symptoms will be tested for MG. Sudden-onset ptosis, diplopia, and dysphagia are classic signs/symptoms of MG. Laboratory studies and a cholinesterase inhibitor test (e.g., Tensilon challenge test) most likely will be done to confirm the diagnosis. Signs/symptoms of Bell's palsy include facial paralysis; the face appears masklike and sags. Signs/symptoms of GBS typically begin in the legs and spread to the arms and upper body. Trigeminal neuralgia is characterized by sharp, intense facial pain that is usually not associated with sensory or motor deficits.

A client is being discharged with paraplegia secondary to a motor vehicle crash and expresses concern over the ability to cope in the home setting after the injury. Which is the best resource for the nurse to provide for the client? >Hospital library >Internet >National Spinal Cord Injury Association >Provider's office

National Spinal Cord Injury Association The best resource for the nurse to provide is the National Spinal Cord Injury Association. The National Spinal Cord Injury Association will inform the client of support groups in the area and will assist in answering questions regarding adjustment in the home setting. The hospital library is not typically consumer-oriented. Most information available in the library is targeted to health care professionals. The Internet is not the best resource simply because of the unlimited volume of information available and its questionable quality. Although the provider's office may have information, the information may not be as comprehensive and current as other options.

The nurse is caring for a client with a spinal cord injury resulting from a diving accident, who has a halo fixator and an indwelling urinary catheter in place. The nurse notes that the blood pressure is elevated and that the client is reporting a severe headache. The nurse anticipates that the primary health care provider will prescribe which medication? >Dopamine hydrochloride (Inotropin) >Methylprednisolone (Solu-Medrol) >Nifedipine (Procardia) >Ziconotide (Prialt)

Nifedipine (Procardia) The nurse anticipates that the primary health care provider will prescribe nifedipine for a spinal cord injury client who has an elevated blood pressure and severe headache. This client is experiencing autonomic dysreflexia (AD). Nifedipine (Procardia), a calcium channel blocker, can be administered to treat AD and lower blood pressure. If AD is not treated, a hemorrhagic stroke can occur. Dopamine hydrochloride (Inotropin) is an inotropic agent used to treat severe hypotension. Methylprednisolone (Solu-Medrol) is a glucocorticoid and is not indicated because it may further increase blood pressure. Ziconotide (Prialt) is an N-type calcium channel blocker on those nerves that usually transmit pain signals to the brain.

A client presents to the Emergency Department from an assisted living facility after a ground level fall with a head strike. The client has a Glasgow Coma Score (GCS) of 12, which is decreased for this client, and has projectile vomiting. What is the priority intervention for this client? >Calling the Stroke Team >Establishing an IV >Positioning the client to prevent aspiration >Preparing for thrombolytic administration

Positioning the client to prevent aspiration Positioning the client while maintaining cervical spine immobilization to prevent aspiration is the nurse's priority intervention. Maintaining a patent airway is essential especially since this client is vomiting. Calling the Stroke Team would not be necessary. Establishing an IV is important for this client but it is not the first priority. If this client was having a stroke, thrombolytics would be contraindicated because of the fall with head strike.

The nurse is administering the intake assessment for a newly admitted client with a history of seizures. The client suddenly begins to seize. What does the nurse do next? >Documents the length and time of the seizure. >Forces a tongue blade in the mouth. >Positions the client on the side. >Restrains the client.

Positions the client on the side. When a newly admitted client with a history of seizures begins to seize, the nurse must turn the client on his/her side. Turning the client on the side during a generalized tonic-clonic or complex partial seizure is indicated because he or she may lose consciousness resulting in potential loss of a patent airway. Documenting the length and time of seizures is important, but not the priority intervention. Both forcing a tongue blade in the mouth and restraining the client can cause injury.

A client with myasthenia gravis is admitted with generalized fatigue, a weak voice, and dysphagia. Which client problem has the highest priority? >Inability to tolerate everyday activities related to severe fatigue >Inability to communicate verbally related to vocal weakness >Inability to care for self-related to muscle weakness >Potential for aspiration related to difficulty with swallowing

Potential for aspiration related to difficulty with swallowing The client problem that has the highest priority for a client with MG is the risk for aspiration due to difficulty swallowing. The potential for aspiration is the highest priority client problem because the client's ability to maintain airway patency is compromised. Although important, an inability to tolerate everyday activities, an inability to communicate verbally related to vocal weakness, and an inability to care for oneself related to muscle weakness are not the nurse's highest priority.

A client with early-stage Alzheimer's disease is admitted to the surgical unit for a biopsy. Which client problem is the priority? >Potential for injury related to chronic confusion and physical deficits >Risk for reduced mobility related to progression of disability >Potential for skin breakdown related to immobility and/or impaired nutritional status >Lack of social contact related to personality and behavior changes

Potential for injury related to chronic confusion and physical deficits The priority client problem related to a client admitted to the surgical unit for biopsy is the potential for injury due to chronic confusion and physical deficits. The most important intervention for interdisciplinary care is safety. Chronic confusion and physical deficits place the client with Alzheimer's disease at high risk for injury. Reduced mobility, skin breakdown, and lack of social contact, although potential problems in this population, are more frequently observed in the long-term setting and not the top priority.

To prevent the leading cause of death for clients with spinal cord injury, collaboration with which component of the primary health care team is a nursing priority? >Nutritional therapy >Occupational therapy >Physical therapy >Respiratory therapy

Respiratory therapy To help prevent death for a client with spinal cord injury, collaboration with the Respiratory therapy team is a priority. A client with a cervical spinal cord injury is at risk for breathing problems including pneumonia and aspiration, resulting from the interruption of spinal innervation to the respiratory muscles. Collaboration with Respiratory therapy is crucial. Collaboration with nutritional therapy, occupational therapy, and physical therapy does not help prevent the leading cause of death in clients with spinal cord injury.

The nurse is caring for a client in the emergency department (ED) whose spinal cord was injured at the level of C7 1 hour prior to arrival. Which assessment finding requires the most rapid action? >After two fluid boluses, the client's systolic blood pressure remains 80 mm Hg. >Cardiac monitor shows a sinus bradycardia at a rate of 50 beats/min. >The client's chest moves very little with each respiration. >The client demonstrates flaccid paralysis below the level of injury.

The client's chest moves very little with each respiration. The most rapid action is needed for a spinal cord injury client injured one hour prior to arrival whose chest moves very little with each respiration. Airway and breathing are always of major concern in a spinal cord injury, especially in an injury near C3 to C5, where the spinal nerves control the diaphragm. Bradycardia and hypotension are indications neurogenic shock due to disruption of autonomic pathways. This will need to be addressed rapidly however airway and breathing are always the top priority. Flaccid paralysis below the level of the injury is to be expected.

The nurse has received report on a group of clients. Which client requires the nurse's attention first? >Adult who is lethargic after a generalized tonic-clonic seizure >Young adult who has experienced four tonic-clonic seizures within the past 30 minutes >Middle-aged adult with absence seizures who is staring at a wall and does not respond to questions >Older adult with a seizure disorder who has a temperature of 101.9° F (38.8° C)

Young adult who has experienced four tonic-clonic seizures within the past 30 minutes After receiving report on a group of clients, the nurse first needs to attend to the young adult client who is experiencing repeated seizures over the course of 30 minutes. This client is in status epilepticus, which is a medical emergency and requires immediate intervention. The adult client who is lethargic and the middle-aged adult client with absence seizures do not require immediate attention. A fever of 101.9° F (38.8° C), although high, does not require immediate attention.

Which client diagnosed with neurologic injury is typically at highest risk for depression? >Older man with a mild stroke >Older woman with a seizure >Young man with a spinal cord injury >Young woman with a minor closed head injury

Young man with a spinal cord injury A young man with a spinal cord injury is at highest risk for depression. Although each individual responds differently, young adults who experience a spinal cord injury and loss of independent movement are more likely to experience depression. Keeping in mind people's differences in personal experiences, the client with a mild stroke without long-term deficits, the client who had a seizure or the young woman who sustained a minor head injury are generally at a lower risk of depression.


Ensembles d'études connexes

Chapter 4 Quiz - Making Decisions

View Set

introduction to psychology, James W. Kalat Chapter 8

View Set

Dosage Calculation 3.0 Pediatric Medications Test

View Set

RE: Types of Loans, Terms, and Issues

View Set

Nature of Insurance, Risk, Perils, and Hazards

View Set